(Solved) EnEc Problems

Download as pdf or txt
Download as pdf or txt
You are on page 1of 198

Chapter 1

Problem (1-1)

A power plant generates electricity from coal. If the price or electricity per unit energy is
seven times the price of coal ($/kJ), and the coal cost represents 50% of the total cost for
generating electricity, determine the minimum physical efficiency, ηph of the plant which
ensures an economic efficiency, ηEc of at least 120 percent.

Solution 1.1:

* ηEc = cost ratio * ηPh

* ηPh = ηEc/cost ratio


𝑠𝑎𝑙𝑒 𝑝𝑟𝑖𝑐𝑒 𝑝𝑒𝑟 𝑢𝑛𝑖𝑡
cost ratio = 𝑐𝑜𝑠𝑡 𝑝𝑟𝑖𝑐𝑒 𝑝𝑒𝑟 𝑢𝑛𝑖𝑡

7𝑥 7
= 2𝑥 = 2

7
ηPh = 1.2 * 2 =0.3429

Physical efficiency of 34.29% or more ensures economic efficiency at least 120%.

Problem 1.2
A 25 MW power plant is designed to generate electricity from fuel oil. The fuel cost is a
function of plant size, thermal conversion efficiency (i.e. heat rate or physical efficiency),
and the plant utilization factor. The fuel cost per year is given by the following
expression:

C * U  8760 hr yr 
f   p
 ph  103 
where
f  annual fuel cos t ($ / yr)
C  plant size in kw
U  plant utilization factor
 ph  physical efficiency
p  price of fuel oil per thousand kwh

Assume that the fuel oil price, p, is estimated as $ 5.3,the fuel cost represents 42%
of the total cost , the plant operates with utilization factor , U=0.55 and a physical
efficiency ,  ph , of 0.38 while the electricity can be sold at 3.7 cent per KWh.

a) Determine the plant’s efficiency.


b) What is the minimum physical efficiency for the plant to be economically
attractive?
c) What is the minimum electrical energy price for the plant to yield a profit equal
to 24% of the fuel cost?

Solution 1.2:

fuel oil price  $5.3 103 kwh  0.0053 $ / kwh


0.0053
Total fuel cos t   0.00126 $ / kwh
0.42

sale price
 ec   ph 
cos t price
a)
0.037 $ / kwh
 ec  0.38   1.1142
0.0126 $ / kwh

b)For the plant to be economically attractive

0.037
1   ph    ph  0.341
0.0126

c) profit  0.24 fuel cos t  electricity price  fuel total


0.0053 0.0126
 0.24   electricity price 
0.38 0.38
0.0053 0.0126
 electricity price  0.24    0.03656 $ / kwh
0.38 0.38

Problem (1.3)
The total cost of manufacturing 450 units of a product is $ 6550.

This cost is increased to $7000 when 500 units are produced.

a) Determine the total cost for producing 950 units and state any assumption you
used.
b) How is the cost per unit distributed, on percentage basis, between fixed and
variable costs, for the production of: 450, 500, 950 units? Any comments?

Solution:
Assume linear variable costs for the given production units:

Q1=450 C1=6550
Q2=500 C2=7000
Q3=950 C3=??
∆𝐶 7000−6550
= = 9 “The slop of the line”
∆Q 500−450

The total cost of the production of 950 units:

𝐶3 − 7000
=9
950 − 500
C3= $11050

The total cost of the production of 0 units:

6550−𝐶0
= 9𝐶0 = $ 2500"Which represents the fixed cost"
450−0

The percentage of the fixed cost for any production units is given by:
𝑓𝑖𝑥𝑒𝑑 𝑐𝑜𝑠𝑡
𝑓𝑖𝑥𝑒𝑑 𝑐𝑜𝑠𝑡 % =
𝑡𝑜𝑡𝑎𝑙 𝑐𝑜𝑠𝑡
𝑡𝑜𝑡𝑎𝑙 𝑐𝑜𝑠𝑡 − 𝑓𝑖𝑥𝑒𝑑 𝑐𝑜𝑠𝑡
𝑣𝑎𝑟𝑖𝑎𝑏𝑙𝑒 𝑐𝑜𝑠𝑡 % =
𝑡𝑜𝑡𝑎𝑙 𝑐𝑜𝑠𝑡

Q1=450 Q2=500 Q3=950

Fixed cost 2500 2500 2500


(6550) = 38.19% (7000) =35.71% (11050) =22.61%

Variable cost 6550−2500 7000−2500 1050−2500


( ) =61.81% ( ) =64.29% ( ) =77.39%
6550 7000 1050
PROBLEM 1.5:
When the price of a certain mineral ore is increased from $15.00 to $21.00 per Ton, the
mining company increases its production activity from 700 to 1,250 tons per week

a) Find the elasticity of supply.


b) Determine the percentage increase, in the amount spent on purchasing this ore,
for each one percent increase in its price.
c) What would be the expected behavior of the mining company when the ore
has a unitary supply?
d) For 1.25 supply elasticity, determine the price change that justifies the above
production increase from 700 to 1,250 tons per week?

SOLUTION 1.5:

a)

Es  Percentagechange (increase)in quantities supplied, based on the larger quantity


Percentage change (increase)in price,based on the higher price

Where Es ≡ Elasticity of Supply

 Q 
 
% change in quantities supplied  QH 
Es  
% change in price based on higher price  P 
 
 PH 

 1250  700 
 
 1250  0.44
Es    1.54
 21  15  0.2856
 
 
 21 

Es = 1.54 which is more than 1, that means the supply is Elastic

b)

The amount spent A = Q * P

The amount spent when the price is $15, is = 700x$15 = $10500

The amount spent when the price is $21, is = 1250x$21 = $26250


The increase in the amount spent = $26250-$10500=$15750

$15750
 100 %  150 %
The percentage increase in Amount Spent is = $10500

The increase in the Price = $21-$15=$6

$6
 100 %  40 %
The Percentage increase in price is = $15

The percentage increase in the amount for each 1% increase in price is


150%
 3.75%
40%

c) If the price of ore increased from $15 to $21 with elasticity of 1, it will increase the
QNEW
production to

QNEW  700
QNEW
1.0 
21  15
21

QNEW  700
0.2857 
QNEW
QNEW
= 980 tons/week

Ph
d)When the elasticity of supply is 1.25, the new price

 1250  700 
 
Es   1250   1.25
 P  15 
 h 
 
 P 
 h 

Ph  15
1.25 ( )  0.44
Ph

Ph =$8.148 + $15 = $23.148


Problem 1.6:
The time in hours, H , required to finish a certain job

,is inversely proportional to the number of workers , n, ,as indicated by


the following expression:

𝐻 = 1.25 + 46.5𝑒 −0.9𝑛

The hourly wage of the worker can be taken as $4.25 .

a)Determine the shortest & the longest period of time for finishing this
job, and the cost in dollars in each case.

b) Find the point of diminishing returns , for increasing the

number of workers ,and the corresponding cost.

c) What is the optimum number of workers you recommend? Why?


What are the factors in the problem which can affect or alter your
recommendation?

Solution:
We have

H=1.25+46.5e−0.9∗n

The shortest period occurs at n=∞ Hmin = 1.25 hrs

∴Cost = ∞

The longest period occurs at n=1

Hmax = 20.155 hrs

∴Cost = 20.1558492*4.25 = $85.66

b)
n(worker) H(hour) Job cost($)

1 20.1554892 85.660829

2 8.9363983 75.9593856

3 4.37505634 55.7819684

4 2.52055309 42.8494026

5 1.76656834 37.5395772

6 1.46002101 37.2305359

7 1.33538817 39.7277981

8 1.28471624 43.6803522

Point of diminishing return occurs at n* =6

H=1.46002101 hrs

Cost = 6*1.46002101*4.25=$37.23

c)

Optimum No. Of workers = n*=6

Because it results in the minimum cost.

The recommendation can be affected by changing:

✓ The factors of the equation(1.25 ,46.5,-0.9)

✓ The hourly wages of the workers.

Problem 1.7:
The demand, Q, for a certain commodity with 2.8 demand elasticity is 60,000 units when it is
offered for $ 22 per unit. If the total unit cost is $ 19.80, determine the optimum selling
price, to the nearest dollar, for this commodity based on the initial demand condition.

Solution 1.7:
ED = (ΔQ/Qh)/ (ΔP/Ph)

Where

ED = 2.8

Qh = 60,000

ΔQ = ((60,000)*(2.8)*ΔP)/Ph

ΔQ=168000*ΔP/Ph

Total profit Quantity demand Profit ($) Selling Price

($) Qh (Ph-19.80) Ph ($)

132000 60000 2.2 22

168626 52695.65 3.2 23

193200 46000 4.2 24

207168 39840 5.4 25

211754 34153.85 6.2 26

208000 28888.89 7.2 27

196800 24000 8.2 28

178924 19448.28 9.2 29

155040 15200 10.2 30

* P = $26 is the optimum selling price since the total profit = $211754 is maximum at this
price.
Problem 1.8:
An engineer can do certain required computations in 5 hours
or he can delegate the work to an engineer aide. If the work is
delegated, it will take 50 minutes to explain the computational
procedure, and 35 minutes to check the results. The actual
calculations will take 6.5 hours to do if done by the aide. If the
engineer receives a monthly salary of $1,800 and the aide
receives $980 per month, determine:
a) Which method you recommend, assuming a working month
of 160 hours.
b) What is the minimum change, for each of the following
items, (taking one at a time) required for reversing your
recommendation:
i) The assistance's salary?
ii) The engineer's salary?
iii) The explanation time?

Solution 1.8 :
Engineer's salary per hour = 1800/160 = $11.25
Aide's salary per hour = 980/160 = $6.125

Method 1”engineer himself”:


Actual calculations cost for work done by engineer = 5(11.25)
= $56.25

Method 2:
Actual calculations cost for work done by the aide = 6.5(6.125)
= $39.8125
50
Explanation cost = (11.25+6.125)* = $14.4792
60
35
Checking cost = (11.25+6.125)* = $10.1354
60

Total cost of work done by the aide =


39.8125+14.4792+10.1354 = $64.4271
➢ So it is recommended that the engineer should do the
work himself.

➢ The total savings due that is equal to :


Cost of work done by the aide - cost of work done by the
engineer himself = 64.4271 - 56.25 = $8.1771

b)
i) The assistance's salary:
*Total cost of work done by the aide must be less than the total
cost of work done by the engineer himself.
Assume aide's salary per hour = S
Total cost for work done by the aide =
85
S(6.5)+(11.25+S)* < 56.25
60
Solving this equation to find the value of S:
S< 5.0921
➢ So the assistant's month salary should not exceed
(5.0921*160) = $814.736

ii) The engineer's salary:


*Total cost of work done by the engineer himself must be
greater than the total cost of work done by the aide.
Engineer’s salary per hour = E
85
(5)E > (E+6.125)* + (6.125)*(6.5)
60
E > $13.7236
➢ So the engineer's monthly salary should be more than
(13.7236*160) = $2195.77

iii) The explanation time:


*Total cost of work done by the aide must be less than the total
cost of work done by the engineer himself.
The explanation time per hour = T
35
(11.25+6.125) + (6.125) (6.5) + (11.25+6.125)T < 56.25
60
T <0.36271 hours

➢ So the explanation time should be less than


(0.36271*60) = 21.76264 minutes
➢ If the explanation time is reduced to less than 21.76264
min. method (2) becomes more attractive
= 21.76264/50 = 43.525%
50−21.76264
➢ Explanation time reduction = ∗ 100%
50
= 56.477%
➢ Explanation time should be reduced by at least 56.477%
of its original value in order to reverse the above
recommendation.
Chapter 3
Problem 3.4:

a) Draw a cash flow diagram for $ 8,700 being loaned out on October 1, 2008 which
will be repaid on July 1, 2016 with a simple interest rate of 11% per year, what is the
total amount paid as interest?

b) In part (a) if the borrower fails to pay any interest until the end of the period and
interest is compounded at the rate as before, determine how much extra interest is
being paid in this case and the reason for this extra amount?

Solution 3.4:

a)
9
The period of the loan is 7 years and 9 months, 9 months = 12 years = 0.75 years.

n = 7 + 0.75 = 7.75 years.

i = 11% per year at simple interest.

Total interest paid:

I = P i n = 8,700 0.11 7.75  $ 7,416.75

7,416.74
Interest paid per year = 7.75 = $ 957

b)

n = 7.75 years.

i = 11% per year compounded interest.

F = (1+i)n

= 8700 (1+0.11)7.75
= $ 19,533.15239

Total interest paid:

I=F–P

= 19,533.15239 – 8700

= $ 10,833.15239

Extra interest paid:

= 10,833.15,239 – 7,416.75

= $ 3,416.40239

He will pay more interest because interest is accumulated until loan is due, that is
mean interest is charged on the total amount owed “principle plus interest”.

Problem 3.5:

a) Differentiate between “annual” and “continuous” compounding.

b) For what periods of time will $7500 have to be invested at a nominal interest
rateof 12% to amount to $10000 using:

(i) Simple interest?

(ii) Annually compound interest?

(iii) Continuously compounded interest?

c) How many years will it take for an investment to double itself if the interest
rate is 15% compounded:
(i) Annually?

(ii) Continuously?

d) An investor wishes to have $ 22,000 available 10 years from now. If $10,000 is


available for investment at the present time, what interest rate on the investment
would be necessary to give the desired amount when the interest is compounded:

(i) Annually?

(ii) Continuously?

Solutions 3.5:
a)
Annual compounding Continuous compounding

Frequency of compounding ( Ǿ ) is unity Frequencies of compounding ( Ǿ ) tend to


infinity.

Interest is calculated once year. Interest is calculated continuously.

The effective interest rate ieff is identical The effective annual interest rate ieff is
with the nominal rate r : given by :

ieff = r , Ǿ = 1 . ieff = er-1 Ø →. ∞

Number of compounding periods are No. Number of compounding periods per year
of year. tends to infinity.

Length of compounding period is one year. Length of compounding period tends to zero.

b)

(i) Simple interest:

I=Pin
I = F – P = 10000 - 7500 = $2500.

I = 12%

n = I / Pi = 2500 / (7500 * 0.12) = 2.7778 years.

(ii) Annually compounded:

General solution:

F=P (F/P i , n )

F =10000 , P =7500 & i =12%

$10000=$7500 ( F\P i , n )

1. Formal solution:
10000 = 7500 (1+0.12) n

ln (10000 / 7500) = n * ln (1+0.12)

n=2.5384 years.

2. Table solution:
( F/P 12 , n ) = 1.3333
From table when i=12% we get :

( F/P 12 , 2 ) = 1.2544
( F/P 12 , 3 ) = 1.4049
( y - y1 ) / ( y2- y1 ) = ( n - n1 ) / ( n2- n1)
(iii) Continuously compounded:

F=P [F \ P r,n]
F= P ern
F=10000 , P=7500 & r=12%
10000=7500* e0..12 n
10000 / 7500 = e0..12 n
Ln(10 / 7.5) = 0.12*n
n=2.397 years.

c)

(i) Annually.
F=P ( F \ P i , n ).
F= P (1+i) n
F=2P & i=15%
2P=P (1+i) n
Ln 2= n * ln 1.15
n =4.96 years.

(ii) Continuously.

F=P [F \ P i , n]
F=P ern
2P=P e0.15n
n=4.62 years.
d)

(i) Annually.

F=P ( F \ P i , n)
F=P (1+i) n
F=22000 , P=10000 & n=10
22 ,000=10,000*(1+i) 10
Ln(22000/10000)=10 ln(1+ i)
(1+i) = e0.078845
i = 8.203%.

(ii) Continuously.
F=P [ F \ P i , n]
F=P ern
F=22000 , P=10000 & n=10 .
22000 = 10000 * e10r
Ln (22000/10000) = 10 * r
r =7.8845%.
Problem 3.6:
a) If the nominal interest rate is 14% compounded annually, find the present-
worth of a $ 9000 receipt due in 3 years, plus an $ 11500 receipt due in 5 years,
plus a $ 7800 debt due in 8 years, plus a $18500 receipt due in 10 years, plus a
$ 12800 debt due in 13 years.
b) If $ 17500 is owed, what equal beginning-of-year payments for 7 years will
discharge the debt when money is worth 11% per year?

Solution 3.6:

a) Cash flow diagram:


P = 9000(P/F 14 , 3) + 11500(P/F 14 , 5) - 7800 (P/F 14 , 8) + 18500 (P/F 14 , 10) -
12800(P/F 14 , 13)

Using table A.20

P = 9000(0.67497) + 11500(0.51937) - 7800 (0.35056) + 18500 (0.26974) -


12800(.18207)

= 6074.73 + 5972.755 - 2734.368 + 4990.19 - 2330.496 = $ 11,973

b) cash flow diagram:

P-1 = P(P/F 11 , 1)

Using table A.17

P-1 = 17500(0.90090) = $ 15765.75

A = P-1(A/P 11 , 7)

Using table A.17


A = 15765.75(0.21222) = $ 3345.8075

Comments on the original solution:

• There is a difference of $10,130.028 between the two solutions, as a result of


taking the amounts at the 8th and the 13th years as receipts instead of
depts..

Problem 3.7:
a) Differentiate between “nominal” and “effective annual” interest rates.

Nominal interest rate : is a Yearly basis that when interest is compounded (


computed and added to the deposit balance ) more frequently than once a year.

Given by:

r=ixc

where:

i = the interest rate per period.

C = the number of periods per year.

(
The effective interest rate 𝑖𝑒𝑓𝑓 ):is the actual amount of interest paid , and the
difference in the compounding frequency results in a real difference in the effective
interest.

For a fixed nominal interest rate (r) , the amount of interest charged (𝑖𝑒𝑓𝑓 ) increases
as the compounding frequency ( ф ) increase.

ieff= f ( r, ф )

where :

ieff = the effective interest rate .

r = the nominal interest rate .


ф= the compounding frequency.

Also it given by:

ieff-ann = I/P

Where:

I = the actual amount of interest paid per year.

P = the principal sum.

And also:
r
ieff = ieff = (1 + c) ͨ − 1

Where:

ieff = the annual effective interest rate.

r = the nominal interest rate.

C = the number of actual compounding periods per year as stated in the original
financial arrangement.

b)What is the effective interest rate that corresponds to a nominal rate of 11.25%
compounded?

i) annually
ii) monthly
iii) continuously

𝑖𝑒𝑓𝑓 = ? r = 11.25 %

(i) Annually :

C=1

ieff = ( 1 + r ) -1 = r =11.25 %
When interest is compounded annually the effective ( annual ) interest rate will be
identical with the nominal interest rate .

(ii) monthly :

for a nominal interest rate ,r, of 11.25 % compounded monthly , the effective
interest rate is given by :

C = 12 period per year

0.1125 12
ieff =(1+ ) −1
12

= 0.1185 = 11.85%.

(iii) continuously:

ieff = еr − 1

= е0.1125 − 1

= 1.1191 – 1

= 0.1191 = 11.91%.

C) Which is better to receive, an interest rate of 1.5 % every month or 9.25% at


the end of every 6 month period? Why?

to compare we first determine the effective semiannual interest rate for each case ,
then they are compared with one another .

Case (i)

When interest is compounded every 6 month , the effective semiannual interest rate :

ieff= ( 1 + i0)m - 1

Where:

i0= the actual interest rate per compounding period.

m= the number of original compounding periods per effective period.


We have:

m= 1 period per 6 month

actual interest rate r/c = i0 = 9.25 %

ieff_semiann= (1+0.0925)1 - 1 = 9.25 %

Case (ii)

for an actual interest rate i0 = 1.5 % every month the effective interest rate :

ieff= ( 1 + i0)m - 1

Where:

m=the original compounding periods per effective period =6

ieff =( 1 + 0.015 )6 - 1

= 1.09344 – 1 = 0.09344 = 9.344 %

Therefore, 1.5% every month is better than 9.25% every 6 month.

d) If 225$ in interest is earned in the first 3 month on an investment of $9,000


when interest is compounded quarterly, what are the nominal and the effective
interest rate?

The equivalent cash flow diagram for the problem is as follows:-


I = 225 $

F = P + I = 9225$

P = F ( P/F i,n
)

9000 = 9225 * 1 / (1+ i )n

9225 /9000 = (1 + i ) n

Since interest compounded quarterly: 3 month ⇒ 1 period

n=1
9225
i = _ 1 = 0.025 = 2.5 %
9000

The nominal interest rate = 2.5 * 4 = 10%

The effective interest rate = (1 + i0)m – 1

m=4

ieff= (1 + 0.025)4 – 1 = 10.38 %

e) If an effective rate of 15% is desired, what must the nominal rate be if


compounding is monthly?

ieff = 15 % , r=?

ieff= (1+ r/c )c – 1

Compounding monthly ⇒ c = 12

ieff = (1 + r/12)12 – 1 = 0.15

⇒1.15 = (1 + r/12)12

⇒1.011715 = 1 + r/12

⇒0.011715 = r/12

∴r = 14.058 %
Problem 3.9:

a) Mr. Smith wishes to accumulate $22,000 in savings account in 5 years. If the bank pays
9% compounded annually on deposits of such size, determine:

1) How much should Mr. Smith deposit in the account as a lump sum now?

2) How much should he deposit annually for 5 years if he decides to make equal end-of-
year payments?

b) What is the equivalent present-worth of $7,500 in obligations at the beginning of each


year for 9 years if interest is 10% compounded continuously?

c) For total yearly payments of $3,900 for 7 years at a nominal interest rate 17%, compare
the compound-amount accumulated at the end of the 7th year if the payments are:

1) end-of-year.

2) beginning-of-year.

3) weekly.

4) continuous.

Solution 3.9:
a)

1)

Cash flow diagram


F=22,000

0 1 2 3 4

i = 9%
P=??

General solution:

P/F i ,n
P  F( )
P/F 9 ,5
 22,000 ( )

Numerical solution:

P/F 9, 5
P  22,000 ( 0.64993)
 $14298.46

2)

Cash flow diagram:


F =22,000

0 1 2 3 4
5 years

A=??

i = 9%

General solution:

A F  A/ F i ,n

 22,000  A/ F 9, 5

Numerical solution:

A  22,000  A/ F
0.16709
9,5

 $3675.48

b)

General solution:

P1  A  P/ A r ,n

 7,500  P/ A 10,1

P0  P1  F/P 10,1

Numerical solution:

P1  7,500  P / A 0.01042 10,10 


 45,078.15
P0  P1  F/P 10 ,1

 45,078.15  F / P 1.10517 10,1 
 49,819.02

Another solution:

P  7,500  7,500 P / A 5.64254 10,9 


 $49819.05

c)

1)

Cash flow diagram:

F=??
i = 17%

0 1 2 3 4 5 6 7 years

A= 3,900

General solution:
F/A i ,n
F  A( )
F/A 17 ,7
 A( )
F/A 17 ,7
 3,900( )

Numerical solution:

F  3,900 F / A 11.77200 17,7 


 $45,910.8

2) Beginning of year:

Cash flow diagram:

F7 F8

0 1 2 3 4 5 6 7 8

A= 3,900
i = 17%

General solution:
F8  F7  F/P i ,n

 F7 
F/P 17 ,1

 45,910.8  F/P 17 ,1

Numerical solution:

F7  45,910.8 from above


F8  45,910.8  F / P 1.17 17,1 
 $53,715.636

3) Weekly:

Cash flow diagram:

F8 = ??

i = 17%

F7 = 45,910.5

General solution:


 1  r m mn  1
F  A0  

 r 

Numerical solution:
 
 1  0.17 52527   1 
F  3,900  
 0.17 
 $52,322.11

Another approach:

3,900
A  $75 weekly
52
r 17
i   0.327%
m 52
n  52  7  364 periods
 1  i n  1 
F  A  

 1 
 1  0.00327364  1 
 75    $52330.8

 0.00327 

4) Continuous:

Cash flow diagram

r = 17% F =??

=$3,900 A 7 years

General solution:
F  A
F/A r ,n 
 

 3,900 
F/A 17 ,7 
 

Numerical solution:

F  3,900 
F/A 17 ,7 
 
 $5,2468.26

Another approach:

 er n 1  e 0.177  1 
F  A    3,900  
 r   0.17 
 $52468.33
Problem 3.10:

A) Determine ten equal end-of-year payments which are equivalent to the following cash
flow pattern when the interest rate is 13% compounded continuously:

END OF YEAR AMOUNT


0 -2700
1 -1200
2 +300
3 +300
4 +300
5 +1300
6 -1000
7 +2300

B) What equal series of payments must be paid every 3-months period into a sinking fund to
accumulate $15,000 in 15 years if the nominal interest rate is 12% compounded quarterly?

C) A house you are interested in is offered for $85,000. A down payment of 20 percent is
required and the nominal interest rate is quoted as 15% compounded monthly. If you want
to make monthly installments for 20 years what will your monthly payment be?
Solution 3.10:
A) First we draw the cash flow diagram which is shown below:

2300
1300
300

0 1 2 3 4 5 6 7

1000
1200
1122
2700
W331

The resultant

Cash flow diagram:

0 1 2 3 4 5 6 7 8 9 10

A=??

r=13%
P0
Compounding frequency= ∞
The above cash flow diagram is equivalent to the addition of the following two cash flow
diagrams:

2000 A=300
1000

0 1 2 3 4 5 6 7 0 1 2 3 4 5 6 7

1500
1300

2700
r =13%

Compounding frequency= ∞

Now we will determine the present worth amount:

P/A 3,7 P/F 13,1 P/F 13,5 P/F 13,6 P/F 13,7

P0=300 [ ] -1500[ ] +1000[ ] -1300[ ] +2000[ ]

To find the following values of these factors we can use TABLE (B) [Discrete payment
continuous compounding] with values of r=13% & Ф =∞ and we can see the below:

P/A 13, 7 P/F 13, 6 P/F 13, 1

[ ] =4.30370 [ ] =0.45841 [ ] =0.8781


P/F 13, 7 P/F 13, 6 A/P 13, 10

[ ] =0.40252 [ ] =0.52205 [ ] =0.19084

So that value of P0 will be:

= [300*(4.3037)] - [1500*(0.87810)] + [1000*(0.52205)] -[1300*(0.45841)] +


[2000*(0.40252)]-2700= (-1994.883$)

Thus value of P0 will equal (-1994.883$) and will be the same for the end of ten year
payment

A/P 13, 10

A=P0 [ ]

Also we can get the value of this factor from the same table which equal (0.19084)

So that A= (1994.883*0.19084) =380.703$

B)

A/F i,n

A=F ( ) and since that (i= 12/4=3%) and (n=15*4= 60) so that:

A/F 3, 60

A=15000( ) and we can get the value of this factor from table (A-9) which
Equal (0.00613) and so:

A=15000*0.00613= $ 91.95$

C)

20% down payment is equivalent to [20 %*( 85000)] =17000$ & the remainder will

Be 85000-17000= 68000$

The cash flow diagram is shown below:

P0=$ 68000


1 2 3 239 240

.....................

A=??

i=1.25%

The nominal interest compounded monthly = (12)*(20) =240 month

i= 15/12=1.25%
A/P i, n

A=P ( )

A/P 1.25, 240

= 68000( )

And since the formula of this factor is [i* (1+i)n] / [(1+i)n -1] so the numerical value of this
factor will be

[0.125*(1+0.125)240 ]/ [(1+0.125) 240 -1] = 0.0131676895

So that A will be [68000*(0.0131676895)]=$ 895.4169


Problem 3.11:

a) Mr. Hazim borrows $2,500 at a nominal interest rate of 12% compounded monthly and
decides to repay the loan in equal monthly payments of $250 starting one month from now.

i) How many payments are required ?

ii) If immediately after fifth payment, Mr. Hazim decides to pay off the loan

completely , determine the amount he must pay.

b) Miss. Nada purchased a car whose cash price is $4,800. She defers payments for 3 months
and makes 30 beginning-of-month payments thereafter. If the interest rate is 1½% per
month on the unpaid balance, determine how much her monthly payments will be.

Solution 3.11:

a)

i) The cash-flow diagram

Equal Payment Present-worth factor :


P = A ( P/A i,n )

= A (P/A 0.01,n)

2500 = A ( 1+i)n– 1 = 250 (1.01)n– 1

i(i+1)n 0.01(1.01)n

(10) (1.01) n = (1.01)n– 1

0.9(1.01) n = 1

(1.01) n = 1.1

n ln(1.01) = ln(1.1)

n = 9.579 months

ii) Equal Payment Present-worth Factor ( for 5 months) :-

Po = A(P/A i,n)

= A(P/A 0.01,5)

= 250 (4.8527) [using the table]

= $ 1213.175

Present-worth factor remained (P‾ ) :-

P‾ = P − P o

= 2500 − 1213.175

= $ 1286.825

Single Payment Future-Amount factor :-

F = P‾ ( F/P i,n)

= P‾ ( F/P 0.01,5)
= 1286.825 (1.051)

= $1352.45

b)

The cash-flow diagram:

-Equal Payment Present-worth factor at relative zero:-

Po = P (F/P i,n)

= P (F/P 0.015,2)

= 4800 (1.0302) = $ 4944.96

A = Po ( A/P i,n)

= Po ( A/P 0.015,30)

= 4944.96 (0.0416)

= $ 205.71
Problem 3.12:
Mr. Husam deposits $2,400 per year into an account which pays a continuous
compounding interest of 9% during the first year after the initial deposit. If the
nominal interest rate increases by 0.5% each year and 7 annual deposits are made,
determine the worth of the fund immediately after the last deposit.

Solution 3.12:
The cash flow diagram:

General solution is:

𝐹1 = A (F/Ai,n) 𝐹2 = 𝐹! (F/Pi,n) + A

𝐹3 =𝐹2 (F/Pi,n ) + A 𝐹4 = 𝐹3 (F/Pi,n) + A

𝐹5 = 𝐹4 (F/Pi,n) +A 𝐹6 = 𝐹5 (F/Pi,n) +A

Numerical solution using formulae

𝐹1 = 2,400 (F/A9,2)
2
(1+0.09 ) − 1
𝐹1 = 2,400[ 0.09
] = 2400 * 2.09

𝐹1 = 5016 $

𝐹2 = 5016 (F/P0.095,1) + 2,400

𝐹2 =5016 (1 + 0.095)1 +2,400

𝐹2 =5016 * 1.095 = 7892.52 $


𝐹3 =7892.52(1 + 0.10)1 + 2,400

𝐹3 = 7892.52 * 1.1 + 2.400 = 11081.772 $

𝐹4 =11081.772(1 + 0.105)1 + 2,400

𝐹4 = 11081.772 * 1.105 + 2,400 = 14645. 35 $

𝐹5 =14645.35 (1 + 0.11)1 + 2,400

𝐹5 = 14645.35 *1.11 + 2,400 = 529369 .17 $

𝐹6 = 529369.17 (1 + 0.115)1 + 2,400

𝐹6 = 529369.17 *1.112 + 2. 400 = 592646.624 $

➢ The worth of the fund immediately after the last deposit 592646.624 $

Problem 3.13:
a) How could you determine a desired equal-payment-series present-worth factor if
you only had a table of?

i) equal-payment-series sinking-fund factor?

ii) single-payment compound-amount factor?

Use the 13% interest table to check your values at n=9 years.

b) How would you determine a desired equal-payment-series sinking-fund factor if


you only had a table of:

i) equal-payment-series present-worth factor?


ii) single-payment compound-amount factor.

iii) single-payment present-worth factor.

Use the 9% interest table to check your values at n=11 years.

Solution 3.13:
a) To determine a desired equal payment series present worth factor, if we have
only a table of:

i) equal-payment-series sinking-fund factor.

Answer:

(P/A i, n) resulted to (A/F i, n) by the following:

(P/A i,n) = 1/(A/F i,n) + i ; i= 13% & n=9 years.

From table A.19:

(A/F i, n)= 0.06487

:.( P/A i, n) = 1/ (0.06478+0.13) = 5.131626212

**Another method:

(F/A i, n) = (P/A i, n) (1+i) n

(P/A i, n) = (F/A i, n)/(1+i)n;

(F/A i, n) = 1/(A/F i,n)

(P/A i,n) = 1 /(A/F i,n) (1+i)n

(P/A i,n) = 5.131568327


ii) single-payment compound-amount factor:

(P/A i,n) related to(F/P i,n) by the following:

(P/A i,n)= (1- (P/F i,n))/i ;

(P/F i,n) = 1 /(F/P i,n)

(P/A i,n)= 1-[1/ (F/P i,n)] ;from Table A.19 (F/P i,n) =3.00404

(P/A i,n) = 1-(1/3.00404) = 5.131653478

0.13

**Another method:

(F/A i,n) = (P/A i,n)(1+i)n

(F/A i,n)= (F/P i,n)-1

(P/A i,n)= (3.00404)-1 = 5.131650227

0.13(1+0.13)9

b) To determine a desired equal-payment-series sinking-fund factor if we have only a


table of :
i-Equal-payment-series present-worth factors:

(A/F i, n) related to (P/A i,n)by the following:

(A/F i,n)= (A/P i,n) - i ;

(A/P i,n)=1/ (P/A i,n)

(A/F i,n)=(1/(P/A i,n))-i

from Table A.19 with n=11years we get:

(P/A i,n)=6.80519

(A/F i,n)=(1/6.80519)-0.09= 0.056946668

ii-Single-payment present-worth factors:

(A/F i,n)related to (F/P i,n)by the following:

(F/A i,n) = (F/P i,n)-1

1 = (F/P i,n)-1

(A/F i,n) i

(A/F i,n) = i

(F/P i,n)-1

(A/F i,n) = 0.09/[2.58043-1] = 0.056946527

iii-Single-payment present-worth factors:

(A/P i,n)=1/(P/A i,n)

(P/A i, n)=[1-(P/F i,n)]  (A/P i,n)= 1

1-(P/F i, n)

(A/P i, n)= (A/F i,n) + i = i


1-(P/F i, n)

(A/F i, n)= i -i

1-(P/F i, n)

(A/F i, n)= 0.05694597

Problem 3.15:
a) Young couples with a 2-year-old son want to save for their son's college
expenses in advance. A savings account that pays 9% compounded annually is
opened for the child. Annual deposits of $ 1500 starting on the child's 3rd
birthday are made. Determine the maximum size of 4 equal withdrawals that
can be made by the child on his 17th, 18th 19th and 20th birthdays.
b) Mr. Husam wishes to establish a fund to provide for the college education of
his daughter,Maha. He wishes the fund to provide $4500 on Maha's 18th , 19th
, 20th and 21st birthdays. Mr. Husam will make the first deposit when Maha is
three and a half years old and will make quarterly deposits thereafter until the
end of Maha's fifteenth year.
If a nominal rate of 12% compounded quarterly can be earned on the fund,
what should the uniform quarterly deposit be?

Solution 3.15:

c) Cash Flow Diagram:


F16 = Ad(F/A i , n)

=1500(F/A 9 , 14)

Using table A.15

F16 =1500(26.01920)=$ 39028.8

Aw = P16(A/P i , n) = 39028.8(A/P 9 , 4)

Using table A.15

Aw = 39028.8(0.30867) = $ 12047.0197

b) Cash Flow Diagram:


ieff = (1+r/c)c-1 where r=12% & c=4

i = ieff = (1+0 .12/4)4 -1 = 0.1255088 = 12.551%

P17 = Aw (P/A ieff , n) = 4500(P/A 12.55 , 4)

(P/A 12.55 , 4) = [(1+ 0.12551)4 -1]/ [0.12551(1+0.12551)4] = 3.00244

P17 = 4500 (3.00244) = $ 13510.96

F15= P17(P/F ieff , n) = 13510.96(P/F 12.551 , 2)

(P/F 12.551 , 2) = 1/(1+0.12551)2 = 0.7894

F15= 13510.96 (0.7894) = $10665.654

i= r/4 =12/4 = 3 % (quarterly)

Ad = F15(A/F 3 , 47) = 10665.654( A/F 3 , 47)

Using table A.9

Ad = 10665.654(0.00996)

Ad= $ 106.13
Problem 3.16:
a) What annual uniform payment series is necessary to repay the following series of
payments when the interest rate is 14% compounded annually:

i) An annual payment of $2,500 increasing at the rate of $250 a year until the end of
the ninth year?

ii)An annual payment of $18,500 decreasing at the rate of $320 a year for a period of
23 years?

b) what is the present – amount ,p , that should be invested now at a nominal interest rate
of 10% compounded annually that will be sufficient for a withdrawal of $3,000 at the end of
the first year and decreasing by $250 every year to a final value of $750 ?

c) Sketch the cash flow diagram of the following payment series . then find out the easiest
way for determining its present – worth :

End of year 1 2 3 4 5 6 7

Amount 0 500 600 700 700 900 1000

Solution 3.16:
Part a:-

We are asked to convert the following into an equal payment series with interest rate14%
compound annually.

I. 𝐴0 = $2500 G=$250 n=9

Cash flow diagram:-


𝐴0 = $2500, G=$250, i=14% A=$3286.58, i=14%

0 1 2 3 4 5 6 7 8 9 0 1 2 3 4 5 6 7 8 9
Years Years

General solution:-

A|G i ,n
𝐴 = 𝐴0 + 𝐺( )

A|G 14 ,9
𝐴 = 𝐴0 + 𝐺( )

Numerical solution:-

Formula:

1 n 
A = 2500 + 250   
 i (1  i )  1 
n

 1 9 
A = 2500 + 250    = $3286.58
 .14 (1  .14)  1 
9

Table: A=2500+250(3.14632) = $3286.58


II. 𝐴0 = $18500 G=$320 n=23yrs

Cash flow diagram:-

𝐴0 = $18500 G= -$320 n=23yrs

........ ........

0 1 2 21 22 23 0 1 2 2 1 22 23
Years Years

General Solution:-

A|G i ,n
𝐴 = 𝐴0 + 𝐺( )

A|G 14 ,23
𝐴 = 18500 − 320( )

Numerical solution:-

Formula:

1 n 
A = 18500- 320   
 i (1  i )  1 
n

 1 23 
A = 18500 - 320    = $16594.4189
 .14 (1  .14)  1 
23

Table:

A=18500-320 (5.95494) = $16594.4189

Part b:-
3000  750
The number of periods =  9 periods
250

Then: n=1+9=10 yrs.

𝐴0 = $3000 G= -$250 n=10yrs

0
1 2 8 9 10

General solution:-

A|G i ,n P|A i ,n
P = {𝐴0 + 𝐺( )}( )

A|G 10 ,10 P|A 10 ,10


P = {3000 − 250( )}( )

Numerical solution:-

Formula:-

1 n  1 10
( A|G 10 ,10
)        30.725
 i (1  i )  1  0.1 (1  .1)  1
n 10

 (1  i ) n  1   (1  .1)10  1 
( P/A 10 ,10
) =       6.1445
 (1  i ) (i )   (1  .1) (.1) 
n 10

P =[3000-250(30.725)] (6.1445)=$12711.573

Tables:-

A|G 10 ,10
( )= 30.725
P|A 10 ,10
( )= 6.1445
P =[3000-250(30.725)] (6.1445)=$12711.573

Part c:-

End of year 1 2 3 4 5 6 7

Amount 0 500 600 700 700 900 1000

Cash flow diagram:-

0 1 2 3 4 5 6 7
Years
P0

= (uniform gradient series starting at second year) -($100 at 5th years)

0 1 2 3 4 5 6 7 0 1 2 3 4 5 6 7
YearsYears

P1
First we find P1:-

A|G i ,n P|A i ,n
P1 = {𝐴0 + 𝐺( )}( )

A|G i ,6 P|A i ,6
P1 = {500 + 100( )}( )

And then:-

P|F i ,1 P|F i ,5
P0 = {𝑃1 ( ) − 100( )
Problem 3.17:

a) A $35000 piece of equipment if suggested for a workshop. It is estimated


that the equipment will result in first-year savings of $3500, a second year
savings of $5000 and savings of $8500 per year thereafter. If the equipment
has zero salvage value at any time and money can be invested at 20%
annually, what is the minimum service life of the equipment that would justify
its purchase?

b) Mr. Mahir borrows $18000 and wishes to repay it with 25 semi-annual


payments. Each payment is to be $125 grater than the previous payment.
Assuming an interest rate of 12% compounded continuously, determine the
size of the last payment made by Mr. Mahir.

Solution 3.17:
a) The cash flow diagram is below:

$8500 $8500
$5000
…..
$3500

1 2 3 4 5 n
i = 20% annually

$35000

p = 3500 (p/f 20,1) + 5000 (p/f 20,2) + 8500 (p/A 20,(n-2)) (p/f 20,2)

= 3500*0.83333 + 5000*0.69444 + 8500*0.69444 (p/A 20,(n-2))

35000 = 6388.75 + 5902.74 (p/A 20,(n-2))

(p/A 20,(n-2)) = 4.84711


Using the interest formula

[(1+i)(n-2) - 1]/i(1+i)(n-2) = 4.8711 = [1.2n-2 – 1]/[1.2n-2 * 0.2]

1.2n-2 (0.9742-1) = -1

n-2 = ln(38.76)/ln(1.2) = 20.06

n = 22.06 years

The minimum service life of the equipment to justify its purchase is 22.06 years.

b)

(2)
(1)
2x+375
x+125
x 2x+125

1 2 25
1 2 3 12.5yrs

18000 18000
First convert the gradient factor to equal payment
factor

(3) A AA

ieff = er - 1

= 0.1133

A = A0 + G (A/G i,n)

From cash diagram (2) G = 250

A = [2x+125] + 250 [1/i – (n/n(1+i)-1)]

= [2x+125] + 1106.122

The present worth of cash (3) is

P = [2x+1231.122] (p/A 11.33,12.5)

The present worth must equal the loan:

[2x+1231.122] = 18000/6.5188

x = 765.06

From cash diagram (1)

The size of the last payment made by Mr. Mahir is

(n-1)G + x = 24*250 + 765.06

= $6765.06
Problem 3.18:

The operating expenditures of a certain project are expected to be $ 3,700 at the end of
first 3 months , $ 4,300 at the end of the second 3 months , and are expected to increase
by $ 600 at the end of each 3 months period thereafter for a total of 7 years. If the interest
rate is 12% compounded quarterly determine:

i) The equivalent quarterly worth of the operating costs (i.e. find the equivalent uniform
cost at the end of each 3 months period)

ii) The minimum amount of money that should be invested after 2 year from now that will
be sufficient to cover all of the operating expenses during the 7 year.

Solution 3.18:

Part (i)
Cash Flow Diagram:

0 1 2 3 28

$3,700
$4,300


$4,900

$19,900
i = 12%/4 = 3% every 3 month.

n = 7 x 4 = 28 period.

 A/G3,28
A = A0 + G  
 

 A/G3,28
  = [1/0.03 – (28/ (1.03^28)-1)] = 11.59298
 

So

 A/G3,28 
A = 3700 + 600  
 11.59298

A = $ 10,655.7893

Part (ii)
First we have to find the future worth (F).


0 1 2 3 28

A = $10,655.7893 A
F = A FlAi,n

F/A3,28 = [(1+0.03)628 – 1] / 0.03 = 42.931

 F/A3,28 
F = $ 10,655.7893  
 42.931 

F = $ 457,462.86

Cash Flow Diagram:

F=$457,462.865

012345678

28


P
P = F P/Fi,n 

= (1+0.03)-20 = 0.5537 P/F3,20

P = 457,462.865

P = $ 253,286.097

Problem 3.19:

Miss Eiman is celebrating her twelfth birthday and will need $5000 on her eighteenth,
nineteenth, twenties and twenty first birthdays for her college education.

Her father agrees to lay aside a cretin amount now and each three month period there
after until she is sixteen years old. These payments form a uniform gradient series where
each payment increases by 10% of the first payment. If money can be invested at a
nominal rate of 12% compounded quarterly what is a father’s first contribution? Sketch a
neat cash flow diagram for the problem.

solution 3.19:
The interest rate compounded quarterly:

12
i  3%
4

At first we have to find P5

The effective interest rate ieff  (1  0.03)  1  0.12550  12.5%


4

(1  i) n  1
( P / A i ,n ) 
i(1  i) n

P5  A( P / A 12.55, 4
)  $5000( P / A 12.55, 4
)  $150120.26

To find P  we substitute the value of P 5 in a single payment present worth- factor

1
(P/ F i ,n
)
(1  i) n

P P5 (P / F i ,n
)

After substituting values on the previous factor:


P  $8312

From the uniform gradient series:

Number of periods is 4*4=16 period

P  A [( A  0.1A )  G[ A / G 3,16 ]][ P / A 3,16 ]

Using tables the values of the factors respectively where found to be:

6.87421, 12.56110

Substitute these factors on the above equation yield that:

8312
A   $354.42646
23.463197

Which represents the father’s first contribute.


Problem 3.21:

a) Starting two years from now, a copper mine is expect to


yield a net cash income of $35,000 per year flowing
continuously for 18 years. If the minimum acceptable rate of
return on investment is 15% per year, what is the maximum
amount that can be economically justified for buying the mine?

b) Convert the series of payments indicated below to a funds-


flow process that extends from the beginning of the 3rd year
until the end of the 11th year , assuming that money can be
invested at 13% compounded continuously ,
X0 = X1 = $500
X2 = X3 = $700
X4 = $1,100 , X5 = $1,300, X6 = $1,500 , X7 = $1,700 , X8 =
$1,900 ,
X9 = $2,100
Where Xt indicates the net cash flow element at the end of
year t.
Solution 3.21:
a)

𝒆𝒓𝒏 −𝟏
P* =Ā [P/͞Ar,n] = Ā[ ]
𝒓𝒆𝒓𝒏
Ā =$35,000
r = 15%
n= 18
P* = $35,000 [P/͞A r,n] = $35,000 * 6.2186 = $217,651.047
P = F * [P/F r,n] = F 𝑒 −𝑟𝑛
P* = F = $217,651.047
r =15%
n= 2
P = $217,651.047 * 0.740818 = $161240.6

b)
➢ For the given series of the payments, the present-worth,
P1 is given as:
P1 = $500 + $500[P/F 13,1] + $700[P/F 13,2] + $700[P/F 13,3] + {$1,100
+$200[A/G 13,6]}* [P/A 13,6] [P/F 13,3] = $5,980.923
➢ For the funds-flow process, the present-worthgiven as:
P2 = P1*[F/P 13 ,2] = P1*(1+i)n = 5,980.923*1.2969
= $7756.659
𝑟𝑒 𝑟𝑛
͞A = P2 *[͞A/P13,9] =P2 * = 7756.659*0.1885 = $1,462.13
𝑒 𝑟𝑛 −1
Problem 3.22:

The Murphy Company manufactures 2.5 million units per year


which sell at $1. 5 less than a competitor’s superior product.
How much can the Murphy Company spend on research for 3
years if the research will improve the product and will succeed
in attaining at least 75% of the $1.5 per unit extra?

Money can be invested at 15% per year, the research costs are
distributed evenly throughout the research period and the
research benefits are expected to start immediately after
completion of the research andwill continue for 7 years.

Solution 3.22:
Research benefits = A2 = 2500000*0.75*$1.5

= $ 2.8125*(10^6)

P3 = A2 [P/A r, n]

P3 = $2.8125*(10^6) [P/A 15, 7]

Using interest formulas:

[P/Ar, n] = ((e^rn)-1)/r(e^rn)

[P/A r, n] = ((e^(0.15*7))-1)/0.15(e^(0.15*7))

[P/A r, n] = 4.33375

P3 = $12.18867*(10^6)
A1 = research costs

A1 = F [A/F r ,n]

A1 = $12.18867*(10^6) [A/F 15,3]

Using interest formulas:

[A/F r, n] = r/((e^(r*n))-1)

[A/F 15, 3] = 0.15/((e^(0.15*3))-1)

[A/F 15, 3] = 0.26394

A1 = $3.2171*(10^6) per year


Chapter 4

Problem 4.1:
The Bright-star Firm pays $ 108,000 for a piece of heavy
equipment. The equipment has a life of 12 years with no
salvage value. After 6 years of use, an overhaul costing $10,800
will be needed. If money can be invested at a nominal rate of
15%, how much must the firm deduct from the equipment
yield by the end of each year over its 12-year life in order to
recover the capital and all other costs associated with the
equipment?

Solution 4.1:
Capital money for equipment =108000$

n =12%

i =15%
Over haul cost =10800$ after 6 years of use

F1

F2
A=?

0 1 2 3 4 5 6 7 8 9 10 11 12

P0 P6

General solution:

F1=P0 (F/P I,n)

And since the formula of the factor is (1+i)n

Numerical solution:

F/P 15, 12

F1=$108000( ) = $108000*(5.35025) =$577827.01

F/P 15, 6
F2=$10800( ) = $10800*(2.31061) = $24981.056

F=F1+F2=$602808.06

General solution:

A/F i, n

A=F ( ) and the formula of the factor is given by:

i/ [(1+i)n -1]

The numerical solution will be:

A/F 15, 12

A=602808.06( ) = 602808.06*(0.0344807)

=$ 20,785.24

Problem 4.2:

Mr. Mahir borrowed $22,000 and agreed to pay $1390 at the


end of each month for 18 months to discharge the debt . If ,
Mr. Mahir was unable to pay the tenth and eleventh
installments , what equal monthly payments thereafter would
be needed to discharge the debt on time ?

Solution 4.2:
Cash flow diagram

A=$1390

1 2 3 4 5 6 7 8 9 18

P9
P =$22,000

P  A( P / Ai ,18 )
22000  1390( P / Ai ,18 )  ( P / Ai ,18 )  15.827338

From tables:

i  1.25%  ( P / Ai ,18 )  16.02960


i  1.5%  ( P / Ai ,18 )  15.6725
Using interpolation:

i  1.25 1.5  1.25



15.827338 16.02960 15.6725  16.02960
i  1.392%
P9  A( P / Ai ,9 )  1390( P / Ai ,9 )

From tables:

( P / A1.25,9 )  8.46238
( P / A1.5,9 )  8.36049
x  8.46238 8.36049  8.46238

1.392  1.25 1.5  1.25
 X ( P / A1.392,9 )  8.4045065
 P9  1390(8.4045065)  $11582.254

9 10 11 12 13 14 15 16 17 18

P9 P11

P11= P9 (F/P 1.392 , 2)= P9 (1+i)n


=11682.264 (1+0.01392)2

=$12009.762

i (1  i ) n

A1 =P11 (A/P 1.392 , 7) = P11 ( (1  i ) n  1 )=12009.762(


1.392 (1.01392) 7
100
1.013927  1 )

=12009.762(0.1509213) =$1812.5297

Problem 4.3:
a) How long will it take to discharge a debt of $25,000 using
beginning-of-year payments of $3,800 at a nominal interest rate
of 13% compounded continuously?
b) If equal payments of $8,500 at the end of each year for 10 years
are needed to discharge a debt of $40,000 , what is the rate of
return assuming:
(i) Annual compounding?
(ii) Continuous compounding?
Solution 4.3:
a) Cash flow diagram
P/A r, n
P - A  A[ ]

 rn
1 e
25,000  3,800  3,800
e 1
r

0.13n
1 e
5.5789 
1
0.13
e
 0.13n ln 0.22548
e  0.22548  n   11.4577 years
 0.13

 To discharge the debt we need 11.46 years.

b) (i) Cash flow diagram


P/A i, n
PA( )

(1i)  1
n

40,000  8,500
i(1i)
n

Taking i = 16%

(1 0.16)  1
10
P/A 16,10
( )   4.83323
0.16(1 0.16)
10

Taking i = 17%

(1 0.17)  1
10
P/A 17,10
( )   4.65860
0.17(1 0.17)
10

 The interest rate is 16% < i < 17%. Thus we use linear
interpolation
4.83323  4.65860 4.70588  4.65860
  i  16.2707%
17  16 i  16

 The interest rate is 16.2707%

b) (ii) Cash flow diagram


P/A i, n
P  A[ ]

 rn
1 e
40,000  8,500
1
r
e
Taking i = 15%
10*0.15
P/A 15,10 1 e
[ ]   4.8004
1
0.15
e

Taking i = 16%
10*0.16
P/A 16,10 1 e
[ ]   4.59973
1
0.16
e

 The interest rate is 15% < i < 16%. Thus we use linear
interpolation

4.8004  4.59973 4.70588  4.59973


  r  15.52897%
16  15 15  r

 The interest rate is 15.52897 %


Problem 4.4:
a) Identify the following interest factors (give their full
names)and use the interest tables to determine their values.
Then check this values using the interest formulae and
determine the percentage error in each case. Are the
percentage errors of the same magnitude? Any comment?

i) [ A/ F ] ii) ( P/ A )
7 ,10.3 7.7 ,5.5

iii) [ A/ G ] iv)
5.5, 7.5
[ A/ F 10.5,13.3
]

b) A man has borrowed $1,500 from the XYZ loan company to


buy a used car and has an agreement to repay the loan
company $500 at the end of each month 3-month period and
$1,000 at the end of the 9th month. Assuming quarterly
compounding of the interest, determine the nominal and
effective interest rates involved in this agreement.

c) Determine the nominal interest rate, r; a certain amount


takes 6 years to double in magnitude. Assuming;

i.) annual compounding

ii. ) continuous compounding.


Solution 4.4:

a) [ A / F 7,10.3 ] This is an equal-payment-series sinking fund


factor for continuous compounding using tables we get the
following value :

[ A / F 7,10.3 ] = 0.07152 & [ A / F 7,11 ] = 0.06252

Assume:

X 1 = n 1 = 10 & Y 1 = 0.07152

X 2 = n 2 = 11 & Y 2 = 0.06252

X = n 3 = 10.3 & Y = ???????

By linear interpolation :
Y  Y1 X  X1

Y2  Y1 X 2  X 1

Y  0.07152 10.3  10

0.06252  0.07152 11  10
Y=0.06882 ≡ The approximate value of the factor .

By using formulae:

er 1
[ A / F r,n ] = e rn  1

e 0.07  1
[ A / F 7,10.3 ] = e 0.07(10.3)  1 = 0.06863 ≡ the exact value

approximate value  exact value


percentageerror  %
exact value

0.06882 0.06863
 %  0.2768
0.06863

Which is approximately 0.3% of error .

ii. ( P / A 7.7 ,5.5


) This is an equal-payment-series present
worth factor

from tables we found :

( )  4.10020
P / A 7 ,5
(P/ A 8, 6
)  4.62288

(P/ A 8, 5
)  3.99271

(P / A 8, 6
)  4.62288

By using linear interpolation :


Y  Y1 X  X1

Y2  Y1 X 2  X 1

Solving for

&( To find
P / A 7,6
) (P/ A 7 , 5.5
)

Y  4.1002 5.5  5

4.76654  4.1002 6  5

Y = 4.43337

And do so for (P/ A ) & (P/ A to find


8, 5 8, 6
) (P/ A 8, 5.5
)

Y  3.99271 0.077  0.07



4.62288  4.43337 0.08  0.07

Y = 4.345467 ≡ approximate value of ( P / A 7.7 ,5.5


)

By using formulae :
 (1  0.077)5.5  1 
(P/ A 7.7 , 5.5
)   5.5 
 0.077(1  0.077) 

 (1  0.077)5.5  1 
(P/ A 7.7 , 5.5
)   5.5 
 0.077(1  0.077)  = 3.50809 represent the exact
value

[ A/ G 6 ,8
]  3.18621

The percentage error is less than -0.2 & this is small error
specially in this case where we use more than one interpolation
.

iii) [ this is a uniform-gradient-series factor for


A / G 5.5, 7.5
]

continuous compounding .

from tables we found :

[ A/ G 5, 7
]  2.80042

[ A/ G 6, 7
]  2.7607

[ A/ G 5,8
]  3.23822

[ A/ G 6 ,8
]  3.18621

Using [ A/G ] & [ A/ G to find [


5, 7 5,8 A / G 5, 7.5
] ]
Y  Y1 X  X1

Y2  Y1 X 2  X 1

Y  2.80042 7.5  7

3.23822 2.80042 8  7

Y = 3.01932

And do so for [ A/ G ] & [ A/ G to find


6 , 7.5 6 ,8
] [ A/G 6 , 7.5
]

Y  2.7607 7.5  7

3.18621 2.7607 8  7

Y = 2.973455

Interpolating between [ A/G ] & [ A/ G to find [ ]:


6 , 7.5 5, 7.5 A / F 10.5,13.3
]

Y  0.036041 0.105  0.10



0.033222 0.03604 0.11  0.10

Y= 2099638 represents the approximate value.

By using formulae:
0.105
[ A/G 10.5,13.3
]
e 1
0.105*13.3

1 0.055
[ A/G 5.5, 7.5
] 
e 0.055
 1 e 0.055*7.5

= 2.997499. represents the exact value


2.99638 2.994799
percentageerror  %  0.037%
2.99638

Which is a very small error .

(iv) [ A/ F This is a funds-flow sinking fund factor .


10.5,13.3
]

By using tables we obtain:

[ A/ F 10,13
]  0.03746

[ A/ F 11,13
]  0.03461

[ A/ F 11,14
]  0.03002

[ A/ F 11,14
]  0.03002

Using [ A/ F ] & [ A/ F to find


10 ,13.3 10.5,13.3
] [ A/ F 10 ,13.3
]

Y  O.O36041 13.3  13

0.33233 0.036041 14  13

Y= 0.036041 The approximate value

Doing so for [ A/ F ] &[ A/ F to find


11,13.3 10 ,13.3
] [ A/ F 10.5,13.3
]

Y  0.036041 0.105  0.10



0.033222 0.03604 0.11  0.10
Y= 0.034637 approximate value.

By using formulae:
0.105
[ A/G 10.5,13.3
]
1
0.105*13.3
e

= 0.034528 represents the exact value


0.034637 0.03452
percentageerror  %  0.31568%
0.034528

it is approximately small error.

b)

$1500  $500( P / A i , 2 )  $1000( P / F i ,3 )

$1500  $500( P / A i , 2 )  $1000( P / F i ,3 )

By trail & error :

At i=13% :right hand side of above equation(R.H.S)=$3.0542

At i= 14% : R.H.S =$2.9966

By interpolating the above:


14%  13% 14%  i

2.9966  3.0542 2.9966  3

i = 0.1394 = 13.94 % quarterly

r = ic = 13.94 % * 4 = 55.76% compounding quarterly


r 0.5576 4
) 1
i eff =(1+ c ) c -1 = (1+ 4 = 0.6854

= 68.54 %

i effquar = (1+0.1394) 1 -1= 0.1394 = 13.94 %

c)

i) annual compounding :

F = P[ F / P r ,n
]

2P = P( F / P i ,6
)

>>> 2 = (1+i) 6

>>> i = 0.1225 = 12025 %

>>> r = ic = i = 12.25 %
ii) continuously compounding :

F = P [F / P r ,6
]

2P = P [F / P r ,6
]

2 = e r (6)

r = 0.1155 = 11.55 %
Problem 4.5:

a) From the interest table, use linear interpolation to determine


the value of the equal- payment-series capital-recovery
factor for 53 periods at 6.25%.

Re-solve the problem using the interest formulae and comment


on any discrepancy between the two values.

b) How long will it take for money to triple if compounded


continuously at a nominal rate of 18%?

c) For funds flowing continuously at a rate of $500 per year


with a nominal interest rate of 8%, sketch the cash flow
diagram, then use the interest tables to determine the minimum
period required to accumulate $5000. check your final answer
by using the interest formulae.

Solution 4.5:
A/P 6.25,53

a- Find : ( )

from the table using Interpolation:


A/P 6 , 55 A/P 7 , 50 A/P 6.25,50

(0.06344) (0.07246)  ( )
A/P 6,55 A/P 7,55 A/P 6.25,55

(0.06254) (0.07147)  ( )
Step1 :-
A/P 6.25,50

To Find : ( )we have:


x1 = i1= 0.06  Y1 = 0.06344

x2 = i2= 0.07  Y2 = 0.07246

x = 0.0625  Y =??

Y-Y1= Y2-Y1

x-x1 x2-x1

Y – 0.06344 = 0.07246 – 0.06344  Y = 0.065695

0.0625 – 0.06 0.07 – 0.06

Step 2 :-

A/P 6.26,55

To find ( )we have:

x1 = i1 = 0.06  Y1= 0.06254

x2 = i2 = 0.07  Y2 = 0.0717
x = 0.0625  Y = ??

Y – 0.06254 = 0.07174 – 0.06254  Y= 0.06484

0.0625 – 0.06 0.07 – 0.06

b)

F=$3

0 1 2 3 ………………… n
r= 18% comp. continuously

P=$1

F/P 18, n

F=P [ ]
F/P 18, n F/P 18, n

F/P = [ ] 3/1 = []

Using Reverse Interpolation :-


F/P 18, 6 F/P 18, 7

[ ][]

X1 = n1 = 6  Y1 = 2.94468

X2 = n2 = 7  Y2 = 3.52542

X = ??  Y=3

X–6 = 7–6  x = 6.095

3 – 2.94468 3.52542 – 2.94468

n = 6.1 years.

Number of period = 6.1years.

c)

F = $ 5000F = $5000 A = $ 500

r =8%

n = ??

0 1 2 ……………… n

F=A [F/A r, n]A = $ 500

5000 = 500 [F/A r , n] ( r= 8 %)


10 = [F/A 8,n
]nominal interest

i- Using Reverse Interpolation:

F/A 8,7 F/A 8,8

[9.38341][11.206]

x1=n1 =7  Y1=9.38341

x2= n2 =8 Y2=11.206

x=n  Y=10

x -7=8 – 7x = 7.338 years

10 – 9.38341 11.206 – 9.38341

:. n= 7.7338 years

ii- Using formula:

F = A [e r n – 1]

10 = e 0.08 – 1  1.8 = e 0.08 n

0.08
0.587787 = 0.08n

:. n = 7.35 years

Step 3 :-

X1 = n1 = 50  Y1 = 0.065695

X2 = n2 = 55 Y2 = 0.06484

X = 53  Y = ??

Y – 0.065695 = 0.06484 – 0.065695  Y = 0.065182

53 – 50 55 – 50

A/P 6.25, 53

() = 0.065182

Using Interest Formula:

(A/P 6.25,53
)= i (1 + i)n = 0.0625 (1.0625)53 .

(1+ i)n – 1 ( 1.0625)53 – 1

 (A/P 6.25,53
) = 0.065119

From the two value we notice that they differ and there is a percentage of error due to
interpolation.
Problem 4.6:
a) If an investment of $27000 provides a continuous flow
of funds at the rate of $550 monthly for 5.5 years
determine the annual interest rate earned by this
investment assuming continuous compounding.
b) The commercial bank offers the following personal loan
plan which is called '' the six percentage plan'' according
to this plan the bank adds 6% to the amount borrowed;
the borrower bays back one-twelfth of this total amount
at the end of each month for a year. If a person borrows
$5000 using this plan, to determine:
i) His monthly payment .
ii) The time interest rate per month
iii) The nominal and effective rates per annum
.
Solution 4.6:

a)

550  1.06
A  $6600 yearly
12

P / A i, n
PA [ ]

P / A i, n
27000= 6600 [ ]

P / A i,5.5
4.090909 [ ]
4.090909

From tables – C – by Looking at The present worth Factor


columns we see
P / A 11,5 P / A 11,6 P / A 11,5.5
[ ] [ ] [ ]
3.84591 4.39226

P / A 12,5 P / A 12,6 [ P / A 12,5.5 ]


[ ] [ ]
3.7599 4.27706

By interpolating:-

n  n1 y   y1

n 2  n1 y 2  y1

Where Y is present worth factor

P / A 11,5.5
y  = [ 4.119085 ]

By interpolation to get i from


P / A i,5.5
[ ]
4.090909

Also

y  =
P / A 12,5.5
[ ]
4.01848

y  y i1  i2

y  y i  i2

4.119085 4.01848 11  12

4.090909 4.01848 i  12

i  11.28%
b)

5000  (1.06)
A  44.667
i) 12

P / A i, n P / A i, n
PA( ) 5000  441.667( )

P / A i, n 5000
( )
441.667

P / A i ,n
( )  11.32074617

ii) From the tables:-


P / A 0.75,12
( )  11.43500
P / A 1,12
( )  11.25510

by linear interpolation

i  0.75 11.32074617 11.43500



1  0.75 11.25510  11.4300 i  0.908%

iii)

r  12  i  12  0.908  10.9%
0.109 12
reff  (1  ) 1
12
 11.46%

Problem 4.7:
a)The blue star company can either buy a certain piece of land
and for outdoor storage of equipment or lease it on a 12-year
lease. The purchase price is $65,000. The annual rent is $4,800
payable at the start of each year. It is estimated that this land
will be needed for only 12 years and will have a saleable value
of $80,000 at the end of the 12-year period.

With the aid of a suitable cash flow diagram, determine the


rate of return, i, that the Blue Star Company will receive by
buying the land instead of leasing it.

b) A person purchases a $6,000 automobile and makes an


initial payment of $1,500 and agrees to pay the remaining
balance in 24 monthly payments of $220 each.

1) Determine the nominal interest rate involved.

2) After 6 months the person decides to increase his


payments in order to shorten his repayments period by 6
months. Assuming a nominal interest rate of 12%.
Determine the required period monthly payment during
the remaining 12 months.

Solution 4.7:
a) Cash flow diagram:

From Figure (1):

F  P (1  i ) n  80,000
F  65,000 (1  i )12  80,000  (1)

0 1 2 3 4 5 6 7 8 9 10 11

12 yrs

Figure (2) A=$4,800

From Figure (2):


F/A i ,n
F  A( )  A (1  i )
F/A i ,11
F  $4,800 ( )  $4,800(1  i )  ( 2)
Solving (1) in (2):
F/A i ,11
$65,000 (1  i )12  $80,000  $4,800 ( )  $4,800 (1  i )
 (1  i )11  1 
$65,000 (1  i )12  $4,800
 
  $4,800i  80,000  4,800
 i 

By trial and error:

Let i  8%

L.H.S  83398.71778

Let i  8.2%

L.H.S  86206.33042

 i between8%  8.2%

By Interpolation:

86206.33042  84800 84800  83398.71778



0.082  i i  0.08
 i  0.08099
 i  8.099%  8 .1 %

 The rate of return i , that the blue star company will receive
by buying the land will be

i  8.1%

b)

1)

Cash flow diagram

$1,500
A
A=$220

1 2 3……………………24

P=$6,000

P/ A i ,n
P  1,500  A( )
P/ A i , 24
6,000  1,500  220 ( )

P/ A i , 24
( )  20.4545

From tables based on n=24 we can find i


From table A.5
P/ A 1.25, 24
( )  20.6243

From table A.6


P/ A 1.5, 24
( )  20.0304

By interpolation:
20.6243 20.4545 20.4545  20.0304

1.25  i i  1.5

 i =1.32147%

The nominal interest rate :


r  12 1.32147
r  15.86%

2)

A0 A0 $1,500

A=$220

1 2 3 6 7 ……………… 18

r = 12%
Let A0 equals the new A that he must pay in order to shorten
his repayments by 6months.

P/ A i ,n P/ A i ,n P/F i ,n
P  1,500  A ( )  A0 ( )( )
P/ A 1,6 P/ A 1,12 P/ A 1,6
6,000  1,500  220 ( )  A0 ( )( )
6,000  1,500  220 (5.79574)  A0 (11.25510)(0.94205)

3224.9966
A0   $304.1626
10.6028
Problem 4.8:

Ms. Eiman approaches the loan tiger agency for $15,000 to be


repaid in 30 monthly installments . the agency advertises
interest at 1.25% per month . However , they proceed to
calculate the size of Ms. Eiman’s payments in the following
manner :

Amount requested = $15,000

Credit investigation = $60

Credit risk insurance =$95

Total = $15,155

Interest : ($15,155)(30)(0.0125) =5,683

Total owed =$20,838


$20,838
Payment : 30 =$694.60

Determine the effective annual interest rate obtained by


the above financing process .
Solution 4.8:

A=694.60 KD

1 2 3 4 5 6 7 8 9 10 11 12 13 14 30 months

P=15,000 KD

Using linear interpolation :

P = 15,000 KD

A = 694.6 KD

n = 30

P = A ( P / A i, n )

15,000 = 694.6 ( P / A i,30 )  21.59516 = ( P / A i, n )

(1  i ) n  1

( P / A i, n ) = i (1  i ) n

(1  i ) 30  1

( P / A i,30 ) = i (1  i ) 30
(1  i ) 30  1

21.59516 = i (1  i ) 30

This equation can be solved for (i) only numerically using


the trial and error procedure :

First trial :

Let i = 2%

(1  0.02) 30  1
0.02(1  0.02) 30 22.395=

Second trial :

Let i = 3%

(1  0.03) 30  1
0.03(1  0.03) 30 19.600414135=

Since the numerical value of the factor is (21.59516) it is


obvious that the required value of i lies between 2% and 3% .

Based on this fact we can make the following additional


trial to narrow the range .
Substituting these values into the following interpolation
formula

Y  Y1 i  i1
Y2  Y1 = i2  i1

gives:

i  0.02 0.199472

0.01 2.7941

i = 2.7139042%
Problem 4.9:

Assume that the present time is November 1985. Mr. Husam


needs $300 by the end of next month. He feels that his needs
are going to increase by $30 each month until the end of
August 1986. Mr. Husam discusses the matter with the
manager of his bank who agrees to meet Mr. Husam’s monthly
needs provided that he pays 15 equal monthly payments of
$315 each starting by the end of September 1986. Assuming
compounding of interest, determine the nominal interest rate
involved in this financial agreement.

Solution 4.9:

Cash flow diagram


A0 = $300

10 11 12 13 14 15 16 17 18 19 20 21 22 23 24 months
0 1 2 3 4 5 6 7 8 9

1985

A = $315
A0 = $300

G = $30

r =??

The two series are equivalent to each other, they have the same
present worth at the same time, let this time be November
1985.

P1 = P2

 A / G i ,n P / A i , n P / A i , n F / P i , n
       
[A0 + G   ]*   =A   *  

 A / G i , 9  P / A i , 9  P / A i, 15   F / P i , 9 
       
[$300 + $30   ]*   = $315   * 

 A / G i , 9  P / A i , 9  P / A i, 15   F / P i , 9 
       
[$300 + $30   ]*   - $315   *  = 0.0
Using trialand error to find i

First: let i = 1%

 A / G i , 9  P / A i , 9  P / A i, 15   F / P i , 9 
       
13.8650   0.914340
[$300 + $30  3.93367  ]*  8.56602  - $315  * =-

412.67421

Second: let i = 1.75%

 A / G 1.75 , 9   P / A 1.75 , 9   P / A 1.75 , 15   F / P 1.75,9


 3.88438   8.26052   13.0929   
0.85544 
[$300 + $30   ]*   - $315   *  =

- 87.294

Third: let i = 2%

 A / G 2 , 9   P / A 2 ,9   P / A 2 , 15   F / P 2 , 9 
       
[$300 + $30  3.86805  ]*  8.16222  - $315  12.8492  *  0.83676  =

9.0295

By linear interpolation:

At: i1 = 1.75% x1= - 87.294

i =? x=0

i2 = 2% x2 = 9.0295
(x-x1)/ (i-i1) = (x2-x1)/ (i2-i1)

(0+87.294)/ (i-0.0175) = (9.0295+87.294)/ (0.0025)

i = 1.976564 %

r = i*c = (1.976564 %)*12 = 23.71876%

i (eff) = (1+r/c)c -1 = (1+0.01976564)12 -1 = 0.264749

i (eff) = 26.4749% per year

Factors formulas:

 A / G i ,n
 
  = (1/i) – n / ((1+i) n – 1)

P / A i , n
 
  = ((1+i) n - 1)/ (i*(1+i) n)

F / P i , n
 
  = (1+i) n
Problem 4.10:

The White Hand Trust Company offers the following personal

loan plan called the "One Percent Plan". For an 18 months

payment period, the plan operates as follows:

The company adds 18th% (1% for each month) to the amount

borrowed, the borrower pays back one-18 of this total at the

end of each month for 18 months. The borrower also has to

pay 0.5% of the total loan as a service charge as soon as he

receives the loan.

Sketch the cash flow diagram for this plan and find its nominal

and effective rates per annum.

Solution 4.10:
Cash flow diagram:

(1/18 + 0.01)P
0.05 P

0 1 2 3 4 5 18

 P/Ai, n 
 
P=A  + A0

 P/Ai,18
 
P = (1/18 +0.01) P   + 0.05 P

 P/Ai,18
 
1 = (1/18 +0.01)   + 0.05

 P/Ai,18
 
0.95 = 0.0655  
 P/Ai,18
 
  0.95 / 0.0655 = 14.504

From interest tables (Table A.8 & Table A.9):

 P/A2,18 
 
  14.992=

 P/A3,18 
 
 = 13.7535

To find i we need to use linear interpolation:

14.9920

14.5040

13.7535

2i 3

13.7535

(i-0.02)/ (14.992-14.504) = (0.03-0.02)/ (14.992-13.7535)


i= 0.02394 = 2.394 %

Nominal interest rate = i * 12= 28.758%

Effective interest rate = ((1+ i) 12 -1) = 32.829 %

Problem 4 .11:
The Loan Shark Company advertises a 9% plan for financing
the purchase of houses. To the amount remaining to be paid
through instalment payment 9% is added for each year in
which money is owned. This total is divided by the number of
months over which the payment are to be made, and the result
is the amount of the monthly payments.

Mrs.Nancy purchases a $75,000 house under this plan, and


makes an initial cash payment of $25,000. she wishes to pay
the balance in monthly payments for 12 years. What will the
amount of each payment be, and what rate of interest does
Mrs.Nancy actually pay
Problem 4.12:

a ) Determine the present equivalent value of $1,500 paid


every 3 months over a period of 5 years in each of these
situations :
i. Nominal interest rate of 18%, compounded annually.
ii. Nominal interest rate of 18%, compounded quarterly.
iii. Nominal interest rate of 18%, compounded monthly.
iv. Nominal interest rate of 18%, compounded continuously.

b) What equal series of payments are necessary to repay a loan


of $45,000 in 5 years, at a nominal interest rate of 15% using
the following payment and compounding frequencies :
1) Monthly payments, with monthly compounding.
2) Annual payments, with semiannual compounding.
3) Semiannual payments, with annual compounding.
Solution 4.12:
a)
i) Compounded annually:-

Fc = 1<Fp = 4
18%
i= = 18%
1
➢ Using the w-shifting method, the equivalent cash-flow
diagram :
A1 = $1500 (F/A i,n) = $1500 (F/A 4.5,4) = $6417.3
P = A1 (P/A 18,5) = 6417.3*(3.12717) = $20068.98
➢ OR:

A*=4A=3000*4=6000
P=A*(P/A i,n)
(1+0.18)5 −1
=6000(P/A 18 ,5
)=6000* =6000*3.12717
0.18(1+0.18)5
=$18763.026
ii) Compounded quarterly :-

Fc = 4 = Fp
basis =3-month period
n = 5*4=20
18%
i= = 4.5% ,
4
(1+i)n −1
P = A (P/A i,n
) = 1500 (P/A 4.5,20) = 1500*( )
i(1+i)n
= $1500 ( 13.00793635) = $19511.905

➢ If we use interpolation which is less accurate :


→ Between (P/A 4,20) and (P/A 5,20) then (P/A 4.5,20)
(13.59030) and (12.46220) then (13.02625)
P = $1500(P/A 4.5,20) = $ 1500 * (13.02625)
= $19539.375

iii) Compounded monthly:-

Approach (1) :
Basis :payment period= 3-month period
n=5*4=20
c
 
 
1  i   1
 c
 
  paymentperiod 3month
 3
ieff= c= int restperiod 1month
18% 3
ieff = ( 1 + ) − 1 = 4.56784%
3∗4
(1+i)n −1
P = A(P/A i,n) = $1500 (P/A 4.5,20) = $1500 ( )
i(1+i)n
= $1500 x 12.93180632 = $ 19397.71

Approach (2) :
Basis: compounding period=one-month
N=5*12=60
i=18/12=1.5%
A = F (A/F i,n)
= $1500(A/F 1.5, 3) = $1500 (0.3284) = $492.6
p=A(P/A1.5,60)=492.6(39.3803)=$19398.7358
Approach (3) :
Treating each payment separately:
P=1500[(P/A1.5,3)+ (P/A1.5,6)+ (P/A1.5,9)+………+ (P/A1.5,60)]=$

iv) Compounded continuously :-


Fc = ∞>Fp = 4
P = A[P/A r,n] = $1500 [P/A 4.5,20]
1−𝑒 −𝑟𝑛
= $1500 [ ] = $1500 * 12.89283977 = $ 19339.259
𝑒 𝑟 −1
b)
i) Monthly payments with monthly compounding:-

Fc = 12 = Fp
𝑖(1+𝑖)𝑛
A = P(A/P i,n) = $45,000(A/P 1.25,60) = $45,000 *
(1+𝑖)𝑛 −1

= $45,000*(0.0238) = $10711

ii) Annual payments with semi-annual compounding:-

Fc = 2 >Fp = 1
Basis: payment period=one year
n =5 year
15
ieff = (1 + ) 2 − 1 = 15.5625 %
2
A = P(A/P i,n) = $45,000 (A/P 15.5625,5)
𝑖(1+𝑖)𝑛
= $45,000 ( ) = $45,000 * 0.3022298294
(1+𝑖)𝑛 −1
= $13603.42

iii) Semi-annual payments with annual compounding:-

➢ Fc = 1 <Fp = 2
➢ Using the w-shifting equivalent cash-flow diagram :

2A = P(A/P i,n)
2A = $45,000 (A/P 15,5) = $45,000 (0.29832) =13424.4
A = $6712.2
Problem 4.13:

a) Miss Nuha plans to deposit $200 weekly into a savings account


starting from the 8th of March until the end of May .determine the
balance in Miss. Nuha's account at the end of May if the account
pays a nominal interest rate of 10% compounded:

1) Weekly.

2) Daily.

3) Monthly.

b) Mr. Mahir makes quarterly deposits of $750 into an account which


pays a nominal interest rate of 18 %. Determine the account
balance immediately after the twelth deposit if the internal rate is
compounded with following frequency, fc:

1) fc = 2

2) fc = 4

3) fc = 12

4) fc = 2 for first year, fc = 4 for the second year ,fc = 12 for the
third year of the loan.

Solution 4.13:
Part a:-
weekly compounding fc=fp i=%10/52=%0.1923

Cash flow diagram:-

0 1 2 3 4 5 6 7 8 9 10 11 12

General solution:-

F|A i ,n F|A .1923 ,12


F = A( ) = 200( )

Numerical solution:-

Formula:-

 (1  i ) n  1   (1  .001923)12  1 
( (F|A .1923 ,12
)        12.128
 i   .001923 

F=200(12.128)=$2425.645

daily compounding fc>fb


 %10 
 
i=  365  =%.027397

cash follow diagram:-

0 1 11 12
........................
Weeks

Since fc>fb, we need to find ieff:-

r .1 7
ieff  (1  ) n  (1  )  1  .001968  .1968%
c 365

General solution:-

F|A i ,n F|A .1968 ,12


F = A( ) = 200( )

Numerical solution:-

 (1  i) n  1   (1  .001962)12  1 
( (F|A .1923 ,12
)        12.13
 i   .001962 

F=200(12.13)=$2426.086
Monthly compounding:- fc<fp

Cash flow diagram:-

0 1 2 3
Months

 10% 
i   .833%
 12 

First we find the new equal payments = 4A = 4*200 = $800

Second we find the future amount F

General solution:-

F|A i ,n F|A .833 ,3


F = A( ) = 800( )

Numerical solution:-

Formula:
 (1  i ) n  1   (1  .00833) 3  1 
( (F|A .833 ,3
)        3.02511
 i   .00833 

F=800(3.02511) = $2420.09

Part b:-

fp=4

fc= 2 =˃ fc<fp =˃ i=%18/2= %9

0 1 2 3 4 5 6 7 8 9 10 11 12
3 Months Periods

Then, we have to do shifting; new payments can be considered each as


double as original payments.

0 1 2 3 4 5 6 7 8 9 10 11 12
3 Months Periods

General solution:-

F|A i ,n F|A 9 ,6
F = A( ) = 1500( )

Numerical Solution:-
 (1  i ) n  1   (1  .009) 6  1 
( (F|A 9 ,6
)        7.523
 i   .006 

F=1500(7.523)=$11285.002

fc=fp=4 =˃ i=%18/4= %4.5

General solution:-

F=A(F/A ,i,n)=750(F/A ,4.5,12)

Numerical Solution:-

Formula:

 (1  i ) n  1   (1  .045) 3  1 
( (F|A 4.5 ,12
)        15.464
 i   .045 

F=750(15.464)= $11598.02

fc= 12 =˃ fc>fp

Here we use the effective interest rate by combining every three months
together

ieff , quartely  (1  imonthly )3  1  (1  .015)3  1  4.57%


General solution:-

F=A(F/A ,i,n)=750(F/A ,4.57,12)

Numerical Solution:-

 (1  i ) n  1   (1  .0457) 3  1 
( (F|A 4.57 ,12
)        15.517
 i   .0457 

F=750(15.517)= $11637.96

at first year fc=2, at second year fc=4, at third year fc=12

Cash flow diagram:

0 1 2 3 4 5 6 7 8 9 10 11 12
3 Months Periods

For fist year we use i=9% A=$1500

For second year we use i=4.5% A=$750

For third year we use i=4.57% A=$750

F⁄A ,9,2)
F1 = {$1500 ( )} = $3135
F⁄A ,4.5,4)
F2 = {$750 ( )} = $3208.64
F⁄A ,4.57,4)
F3 = {$750 ( )} = $3211.99
Net Cash Flow Diagram:

0 1 2 3 4 5 6 7 8 9 10 11 12
3 Months Periods

F1 F2 F3

General solution:-F = F3 + F2( F⁄P , 4.57,4)


)+
F⁄P , 4.5,4) F⁄P , 4.57,4)
F1 ( )( )

Numerical Solution:-

F = 3211.9 + 3208.64(4.2826) + 3135(4.2826)(4.278)= $74389.53


Problem 4.14:

The Mercy Savings and Loan Association advertises that it pays 16


interest compounded quarterly.

a) If Mr. Husam deposits $ 9000 now and plans to withdraw it


after five years, how much would his account be worth at that
time?
b) If Mr. Husam decides to deposits $ 18000 every year for 5
years, determine the amount accumulated in his account at the
end of the fifth year.
c) Suppose that Mr. Husam makes monthly deposits of $ 150 for
the 5-year period. How much could be withdrawn at the end of
the period.

Solution 4.14:
i = 16% compounding quarterly

fc = 4
a) p = $ 9000 , n = 5 years , f=?

Cash flow diagram is below:

F=??

0 1 2 3 4 5
P=9000$
i =16%

i = 16/4 = 4% every three-month period

compounding period = 3-month period

number of interest period, n = 5*4 = 20 periods

interest rate per period, i = 4%

f/p 4,20

f=p( ) = 9000 * (2.19112) = $ 19720.08

b) annual payment A = $ 1800

n = 5 years
adopting the payment period as the basis for calculation :

ieff = (1+ i)c - 1

where:

i = interest rate per original compounding period which is three month


period.

= 16/4 = 4%

c = number of compounding periods per effective period (one year)

c = one year period per/3-month period = 12/3 = 4

ieff = (1+4/100)4 -1 = 16.985%

Cash flow diagram is below:


0 1 2 3 4 5 years

A=1800 A AA

i = 16.985% annually

f/A i,n

F=A( )

Using interest tables:

F = 1800 * 7.01232 = $ 12622.18074

c) A = $150 monthly , n = 5 years

we have three periods each period is with three payments.

Using W-D shifting method:

0 1 year
A AA
Cash flow diagram is below:

F = ??

1 2 3 4 5
20

3A 3A3A

3A = 3* 150 = $450

F = A (f/A 4,20) = 450 * 29.7787

= $13400.415
Problem 4.15:
An engineer is planning to retire in 30 years. He wishes to deposit a
regular amount every 3 months until he retires so that beginning one
year after his retirement, he will receive annual payment of $ 10,000 for
the next 20 years. How much must he deposit every 3-month period
assuming that the nominal interest rate is 8% compounded:

a) Quarterly?

b) Monthly?

c)Annually?

Solution 4.15:

a) compounded Quarterly:
ieff-annually= (1+r/c)c – 1 where r = 8%, c=4

ieff-annually= (1+0.08/4)4 – 1= 8.243%

P*= Aw(P/A i,n)=

P*=10,000(P/A 8.243,20) =10,000[(1+i)n-1/i(1+i)n]

=10,000[(1+0.08243)20-
1/0.08243(1+0.08243)20]=10,000*9.64311=$96,431.16

F*=P*

Ad=F*(A/F i,n) where i=%8/4=%2 & n=30*4=120

=96,431.16(A/F 2,120)=96,431.16[i/(1+i)n-1]=
96,431.16[0.02/(1+0.02)120-1]

=96,431.16*0.002048=$197.5

b) Compounded Monthly:

ieff-annually= (1+r/c)c – 1 where r = 8%, c=12


ieff-annually= (1+0.08/12)12 – 1= 8.299%

P*= Aw(P/A i,n)=

P*=10,000(P/A 8.299,20) =10,000[(1+i)n-1/i(1+i)n]

=10,000[(1+0.08299)20-
1/0.08299(1+0.08299)20]=10,000*9.6035=$96,034.82

F*=P*

ieff-quarterly= (1+r/c)m – 1 where r = 8%, c=12


m=3

ieff-quarterly= (1+0.08/12)3 – 1= 2.0134%

Ad=F*(A/F i,n) where i= ieff-quarterly & n=30*4=120

=96,034.82 (A/F 2.0134,120)= 96,034.82 [i/(1+i)n-1]

= 96,034.82 [0.020134/(1+0.020134)120-1]

=96,034.82*0.002026=$194.598

c) Compounded Annually:
P*= Aw(P/A i,n)

P*=10,000(P/A 8,20)

=10,000[(1+i)n-1/i(1+i)n]

=10,000[(1+0.08)20-
1/0.08(1+0.08)20]=10,000*9.818=
$98,181.47

F*=P*

Using W-S shifting technique

A*=4Ad

A*=F*(A/F i,n) where i=8% & n=30

=98,181.47 (A/F 8,30)= 98,181.47 [i/(1+i)n-1]

= 98,181.47 [0.08/(1+0.08)30-1]
=98,181.47*0.008827=$866.69

Ad=A*/4=$216.6724

Comments on the original solution:

• In a) answer difference is due to round-off error in ieff


• In b) there are errors in calculating the equal-payment
series present worth factor (P/A 8.299,20)
• Modifications in cash flow diagrams are carried out.
• In general as the compounding frequency decreases the
sinking-fund (A) increases.
Problem 4.16:
An increasing annual uniform gradient series begins at the end of the
third year & continues until the end of the twelfth year. What is the
value of the gradient ,G, that makes the gradient series equivalent to a
series of equal end-of -month payments of $800 for 5 years followed by
end-of –quarter payments of $2500 for 3 more years at a nominal
interest rate of 16% compounded quarterly?

solution 4.16:

The equivalent cash flow diagram for the problem is as follows:-

Nominal interest rate r = 16%

i=16%/4= 4%
Period of comparison ,n = 12years=12×4=48 quarters

$800 monthly = 3×$800 =$2400 quarterly

n1= 5 years= 5×4=20 quarters

F1 = 2400(F/A 4,20)(F/P 4,28) = 2400×29.77781×2.9987

= $214309.64

n2=3 years=3×4=12 quarters

F2 = 2500(F/A 4,12) (F/P 4,16)= 2500×15.258×1.8730

= $70357.6135

FT = F1 + F2 = $284667.2535

For the gradient series:-

effective interest rate = ieff=(1+0.04) 4 -1 ≈ 17%

A= Ao+G(A/G 17,10), but Ao= G

A= G (1+(A/G 17,10))
FT = A (F/A 17,10)= G (1 + (1+(A/G 17,10))× (F/A 17,10) =

FT = G×4.2555×22.393

$284667.2535 =G×4.2555×22.393

G=$298.73
Problem 4.17:
Ms. Eiman borrows $10000 and repays the loan in five equal annual
payments. The interest rate for the first year of the loan is 8%
compounded annually for the second year is 10% compounded annually
and for the third year is12% compounded annually.
However, for the fourth and fifth year quarterly and monthly,
respectively. Determine the size of the equal annual payments made by
Ms. Eiman.

Solution 4.17:
The cash flow diagram for the problem is shown below:

Firstly "ieff " must be calculated for the 4th and 5th periods so as to
reduce the original cash flow diagram shown above to the next one
shown below.

i4 eff = (1+i4 )c -1 ;c=4 i4 eff = (1+0.03)4 -1

i4 eff = 12.55%
i5 eff = (1+i5 )c -1 ; c = 12 i5 eff = (1+0.01)12 – 1

i5 eff = 12.7%

To calculate the equal annual payments "A" from the present worth "P",
the accumulated future amount after the5th period "F5 " must be
calculated from the present worth "P"; because of the inequality of the
interest rates per period.
F1 = P (F/P 8,1) (F/P 8,1) = (1+0.08)1 =
1.080

F1 = 10,000 (1.080) = $10800

F2 = F1 (F/P 10,1) (F/P 10,1) = (1+0.1)1 =


1.100

F2 = 10800 (1.100) = $11880

F3 = F2 (F/P 12,1) (F/P 12,1) = (1+0.12)1 =


1.12

F3 = 11880 (1.12) = $13305.6

F4 =F3 (F/P 12.55,1) (F/P 12.55,1) =


(1+0.1255)1 = 1.1255

F4 = 13305.6 (1.1255) = $14975.586

F5 = F4 (F/P 12.7,1) (F/P 12.7,1) =


(1+0.127)1 = 1.127

F5 = 14975.586 (1.127) = $16874.865


The interest rate per year which the present worth "P" should be invested
in; in order to accumulate amount of "F5 " at the end of the 5th period
should be calculated.

F5 = P (F/P i,n) 16874.865 = 10,000


(1+i)5

(1+i)5 = 1.6874 5 ln(1+i) = ln(1.6874)

ln(1+i) = 0.523/5 =0.1046 (1+i) = eo.1o46 = 1.1103

i = 0.1103% annually

Now it’s easy to calculate the end-of-year payments "A" at an interest


rate of “0.1103% annually” required to accumulate the future amount
"F5 " which is equivalent to that from the present worth "P".

A = F5 (A/F i,n )
i 0.1103
(A/F i,n ) = = = 0.1605
[(1+i)n −1] [(1+0.1103)5 −1]

A = 16874.865 * 0.1605 = $2708.416

➢ The size of the equal annual payments made by Ms. Eiman equal to
$2708.416
Chapter 5

Problem 5.1:
b) The Golden Start Company purchased 3 types of machines: X,Y & Z three years ago for its
production activity, The first cost, P,service life, n, salvage value, F, and depreciation model
for each machine are indicated in the table below:

Machine X Y Z
Item
First cost (S) 18,000 25,000 13,000
Service life (years) 12 15 10
Salvage Value ($) 3,500 5,000 3,000
Depreciation Model SYD DB SL

Sketch Btvs "t" for each machine then determine:

i)The depreciation charges for each machine during next year i.e D4 .

ii)The double-declining-balance model rate, R, for each machine.

iii)Use the double-declining-balance model to determine the salvage value of each machine.

Solution 5.1
i)For machine X:
D4=[2(n+1-t)/n(n+1)]*(P-F)=2(12+1-4)/12(12+1) ($ 18,000-$ 3,500)= $ 1673.077

For machine Y:

R=1-n√F/P =1-15√5000/25000 = 0.10174

Where 1/n*≤R≤2/n*

n*=n(P/(P-F))=$15.000*$25,000/($25,000-$5,000)=18.75

Rmin=1/n*=0.0533

Rmax=2/n*=0.10666

D4=R(1-R)t-1.P= $1843.4778

For machine Z :

D4=P-F/n=$ 13,000-$ 3,000/10=$1000

Depreciation Machine Z Machine X Machine Y


by Time t St-Line SYD Model Declining-Balance Model Bt($)
(years) Model Bt($) Bt($) R=0.10174
1 $ 1000 2230.8 2543.5
2 $ 1000 2044.9 2284.72
3 $ 1000 1859.0 2052.276
4 $ 1000 1673.1 1843.4778
5 $ 1000 1487.2 1655.9
6 $ 1000 1301.3 1487.4
7 $ 1000 1115.4 1336.1
8 $ 1000 929.5 1200.2
9 $ 1000 743.6 1078.1
10 $ 1000 557.7 968.4
11 - 371.8 869.9
12 - 185.9 781.4
13 - - 701.9
14 - - 630.5
15 - - 566.3

ii) Rmax=2/n*=2(P-F)/nP
For machine X :

Rmax=2/n*=2(P-F)/nP=2($ 18,000-$ 3,500)/12*$ 18,000=0.13425

For machine Y :

Rmax=0.10666

For machine Z :

Rmax=2/n*=2(P-F)/nP=2($ 13,000-$ 3,000)/10*$ 13,000=0.15384

iii) SV=Bn=(1-Rmax)n.P
For machine X :

SV=B12=(1-0.13425)12.$18,000=$ 3191.40936

For machine Y :

SV=B15=(1-0.10666)15.$25,000=$ 4604.598

For machine Z :

SV=B10=(1-0.15384)10.$13,000=$ 2446.066
Problem (5.2):
(a) Show that for sum-of the years digits model of deprecation the book value at the
end of the year "t" ,Bt, is given by :

Bt= f+ (P-F)(n-t/n)(n-t+1/n+1)

Where

P= first cost

F= salvage value

n= service life

(b) the first cost of an asset is 2500kd and its salvage value after 10 years is 600kd .if the
asset deprecations according to the sum-of-the – years –digits model .sketch its book value
,Bt, at the end of each year during the first five years of its service life .

(c) An asset has a first cost of $900000 an estimated life of 15 years, and an estimated
salvage value of 15% of first cost

1- Using the straight –line model, fine the annul depreciation charge expressed as a
percentage of first cost and the book value at the end of the 10 years .

2- Using the sum-of-the-years-digits model determine the depreciation charge during the
7'Th year and book value at the end of 10 years

Solution:

(a) According to this method :

The deprecation charge during any year ,t, is given by:

Dt = n+1-t (P-F)/∑ j …………..1

j=1

The sum –of-the-years-digits for any number of year's n, can be computed from the
expression:
n

∑ j =1+2+3+…….+ (n-1)+n = n(n+1)/2 ……….2

j=1

From Eq 1 and 2:

Dt =(n+1-t)(p-f)/n(n+1)/2 …………3

The book value at the end of any year t given by:

Bt = p-∑ Dj ………..4

j=1

Substitute Eq 3 into Eq 4 :

Bt = p-[(P-F)/n(n+1)/2]*∑(n+1-t) …….5

t=1

BUT:

∑(n+1-t) = n+(n-1)+……..+(n+1-t)

t=1

= [1+2+3+…..n]-[1+2+3+ …+(n-t)]

= n(n+1)/2 –(n-t)(n-t+1)/2 …………..6

Substitute Eq(6) into Eq(4):

Bt=p-(P-F)/n(n+1)*[n(n+1)/2 –(n-t)(n-t+1)/2]

Bt = p-(p-f)+(p-f)*(n-t)(n-t+1)/n(n+1)

So

Bt = f+ (p-f)[n-t/n][n-t+1/n+1]………….#
(b) By using the equation of the sum-of-the-years-digits we get Bt of the first five years
as follow:

Bt = f+ (p-f)[n-t/n][n-t+1/n+1]

Given:

n = 10

F =salvage value=$600

P= the first cost =$2500

Then:

B0 = p = $2500

B1 =600+(2500-600)(10-1/10)(10-1+1/10+1)

B1 =600+1900*9*10/10*11= $ 2154.545

B2 = 600 + 1900*(10-2/100)*(10-2+1)/11 = $ 1843.636

B3 =600+1900(10-3/10)*(10-3+/11) = $ 1567.27

B4 =600+1900(10-4/10)*(10-4+1/11) = $ 1325.455

B5 =600+1900(10-5/10)*(10-5+1/11) = $ 1118.182

(c) Given:
P = first cost =$900000

n = estimated life =15 years

(1) This model assumes that the value of an asset decreases at a constant rate according to
this model:

Dt = (p-f)/n =constant

Where:

Dt = the deprecation charge during any year

P = the first cost of the asset

F = the estimated salvage value.

n= the estimated service life of the asset

f= B10 =%15 of the first cost (given)

f= 15*900000/100=$135000

Then to expressed Dt as a percentage of first cost and the book value at the end of the 10
years:

Bt = p- ∑ Dt =p-t [(p-f)/n.]

t=0

From EQ (*):

B10 = P- 10 [Dt]

Then:

Dt = p-(B10)/10 .....#

Note that: Dt for this model is constant and its equal

Dt = (900000-135000)/15 = $ 51000
(2)Using sum-of the-years-digit model to get:

D7 & B10

As we proof in part (a)

Dt = (n+1-t/n(n+1)/2) (p-f)

D7 = (15+1-7) (900000-135000) = 7978500 = $ 6648705

15(15+1)/2 120

Also

Bt = f + (p-f) (n-t/n) (n-t+1/n+1)

B10 =135000 + (900000-135000) (15-10) (15-10+1)

15 15+1

B10 = 135000 + 765000 (5/15) (6/!6) = $ 230625


Problem 5.3
A new asset is purchased for $1,200 and is estimated to have a life of 10 years and a scrap
value of $200 at the end of that time.

A) The book value at the end of the sixth year using straight line model and the sum-
of -the -years - digits model.

B) Determine the declining-balance rate,(R) that results in the same book value asthat
given by the sum-of- the-years-digits model.

Solution 5.3:
P=$1200

F=SV =200

n= 10 years

a)

(i) Using straight line model.

Bt = P – t * (P-F) / n

B6 = 1200 - 6*(1200-200) / 10

=$600.

(ii) For sum –of –years-digits model:

Bt = F+ (P-F)[(n-t) (n-t+1) / n(n+1) ]

B6 =200+(1200-200) [ (10-6)*(10-6+1) / 10(10+1)]

=$381.818

b)

Using the declining - balance model.

^t
Bt = (1-R) *p

t
R = 1 – √( Bt / P ) (the end of the sixth` book value was used)

6
R =1- √ (381.818 / 1200)
=0.174

Problem (5.4)
a) Define mathematically“declining-balance model of depreciation”
b) Prove that for its model

𝐵𝑡 = (1 − 𝑅)𝑡 *P

Where:
𝐵𝑡 ≡book value at end of year "t".
P ≡first cost of the asset.
R ≡model rate.

c)Determine the book value of an asset at the end of the 5th year if it has a first cost
of $20,000 and a salvage value of $5000 after 9 years and it depreciates according to
the balance model.

Solution:
a) The declining balance model of deprecation assumes that an asset decreases in
value at a faster rate in the early portion of its life.

Mathematically this model is represented by the following equation:

𝐷𝑡 =R.𝐵𝑡−1
Where:
𝐷𝑡 ≡the depreciation charge during any year
𝐵𝑡−1 ≡the book value at end of the yr, (t-1) or at the beginning of the yr t.
R ≡the depreciation rate which is affixed percentage or a fraction constant

b) From value time equation


𝐵𝑡 =𝐵𝑡−1–𝐷𝑡 ……. (1)

Substituting for "𝐷𝑡 "from equation (1)


𝐷𝑡 =R.𝐵𝑡−1

𝐵𝑡 = 𝐵𝑡−1 – R * 𝐵𝑡−1 = (1-R) 𝐵𝑡−1 …..… (2)

Since the book value of the asset decreases through time so does the size of the
depreciation charge
1 1
As𝐵𝑡 α 𝑡 , 𝐷𝑡 α 𝑡

Equation (2) is recursive relation that relates "𝐵𝑡 " to "𝐵𝑡−1 " for various values of t;
Equation (2) gives:

𝐵1 = (1-R) 𝐵0= (1-R) P

𝐵2 = (1-R) 𝐵2= (1-R)2 * P

𝐵3= (1-R) B2 = (1-R)3 * P

From above equation we can prove that:

𝐵𝑡 = ((1 − 𝑅)𝑡 P

a) The book value at t=5 years

𝑃 = 20,000 $ 𝐹 = 5000 $ 𝑛 = 9 𝑦𝑒𝑎𝑟𝑠

𝑃 20,000
𝑛∗ = 𝑛 [ ] = 9[ ] = 12 𝑦𝑒𝑎𝑟𝑠
(𝑃 − 𝐹) (20,000 − 5,000)
𝑡 𝐵
𝑡
𝑅=√
𝑃

1 2
≤ 𝑅 ≤ , 0.083 ≤ 𝑅 ≤ 0.166
𝑛∗ 𝑛∗
𝑤ℎ𝑒𝑛 𝑡 = 𝑛 = 9

9 5,000
𝑅 =1− √ = 0.142
(20,000)

𝑅𝑚𝑎𝑥 = 0.166

𝑅𝑚𝑖𝑛 = 0.083

𝐵𝑡 = ((1 − 𝑅)𝑡 P

25000

20000

15000

Series1
10000

5000

0
0 2 4 6 8 10

declining-balance model
R max=0.166 𝑅=0.142 R min =0.083
𝐵5 = 20,000(1 − 0.166)5 = 20,000(1 − 0.142)5 = 20,000(1 − 0.083)5
=8069.753 $ =9258.747 $ =12968.109 $

𝐷𝑡 = 𝑅(1 − 𝑅)𝑡−1 ∗ 𝑃

declining-balance model
R max=0.166 𝑅=0.142 R min =0.083
𝐷5
= 0.166(1 − 0.166)4 ∗ 20.000 = 0.142(1 − 0.142)4 ∗ 20.000 = 0.083(−0.083)4 ∗ 20.000
=1606.20 $ =1539.1$ =1173.77$

Problem 5.5:

a) When Double-Declining-Balance Model is used the book-


value of an asset, at the end of the 5th year, was found to be
$4,260. If the first cost of the asset is $13,000, determine its
book-value at the end of the 3rd year.
b) If the Straight Line Model is used, what would be the book-
value of the asset at the end of 5th year.

c) Sketch on the same graph Bt vs. “t” for the two models and
indicate all numerical values, you obtained in previous parts on
your graph.

d) When the two models predict the same book value? i.e.
intersection point on your graph.
Solution 5.5:
a)
Given: B(t) = B(5) = $4,260
First cost (P) = $13,000
➢ For Decline-Balance Model:
Bt = (1-R)t *P …… (1)
➢ Where:
t : time
Bt: book value at time (t)
R :depreciation rate constant
P : first cost or principal value

➢ For DDB: R=Rmax


𝑡 𝐵𝑡 5 4260
R= 1 - √ =1- √ = 0.2
𝑃 13000
B3 = (1 - 0.2)3 * 13000= $ 6,656

b)
➢ From part (a):
2 𝑛𝑃 2(𝑃−𝐹)
Rmax= 0.2 = =2÷ =
𝑛∗ (𝑃−𝐹) 𝑛𝑃
(𝑃−𝐹)
→ = 1,300
𝑛
➢ In straight-line model:
𝑡(𝑃−𝐹)
Bt = P- ...….(2)
𝑛
= 13,000 – 1,300*t
B5 = 13,000 – 1,300*5 = $6500

c)
➢ In Microsoft Excel using the following data:

T Bt Bt
0 13000 13000
2 8320 10400
4 5324.8 7800
6 3407.872 5200
8 2181.03808 2600
10 1395.864371 0
12 893.3531976 -2600

14000
12000
10000
8000
6000
DDB
Bt

4000
SL
2000
0
-2000 0 5 10 15

-4000
t

d)
➢ To find intersection between two models, equalize
equation (1) &(2):
DDB: Bt = (1-Rmax)t *P …..(1)
𝑡(𝑃−𝐹)
SL: Bt = P- = 13,000 – 1,300*t …..(2)
𝑛

(1-0.2)ti*13,000= 13,000 – 1,300*ti


→ F(t) = (0.8)ti-(1-0.1*ti) = 0

➢ From graph. we can see that 8<ti<10 :


F(8) = -0.03222784
F(9) = 0.034217728

➢ By interpolation:
𝑓 (𝑡) − 𝑓(𝑡1) 𝑡 − 𝑡1
=
𝑓(𝑡2) − 𝑓(𝑡1) 𝑡2 − 𝑡1

➢ We find that :
→ ti = 8.48502618years
Problem 5.6:
A certain heat-exchanger costs $45,000 and it has an estimated life of 10 years with a
salvage value of $10,000 at the end of the 10th year.

A) Determine the book value , Bt , of this heat-exchanger at the end of the5th year using
the following depreciation models :
i) Straight-line model
ii) Sum-of-the-Year-Digits Model's Model
iii) Double-declining-Balance, DDB , Model

Sketch Bt vs. t in each case on the same graph

B) What is the rate, R, that should be used with the Declining-Balance Model in order to
predit the same book value B5 as the SYD model?
Is it possible to use this rate for tax purpose? Why?

What will be the salvage value at the end of the 10th year if this rate, R, is used?

C) Which depreciation model do you recommend for your company? Why?

Solution 5.6:
PART (A):

Given

P = $45,000 F = $10,000 n = 10 years t = 5 years

[i] Straight -Line Model:

For Straight- Line Model, the Book value is given by,

t
Bt  p  ( P  F )
n

where:

Bt= the Book value at the end of year t.

P = the first cost of the asset.

F = the estimated salvage value.

n = the estimated service life of the asset.


-The Book value at the end of the 5th year is

5
B5  $45,000  ($45,000  $10,000) = $27,500
10

[ii] The Sum-of-the-Year-Digit Model:

The Book value for SYD model is

( n  t ) ( n  t  1)
Bt  F  (P  F )
n n1

(10  5) (10  5  1)
B5  $45,000  ($45,000  $10,000) = $19,545.5
10 10  1

[iii] The Double-Declining-Balance Model:

Book value is

Bt  (1  Rmax ) t P

Where:

Rmax≡ the maximum depreciation rate = the double rate of the SL Model and is given by

P
n*  n
PF
Where:

n* ≡ the limiting value of the service life , n, for which Bt becomes zero using the straight
line model.

2
RDDB = Rmax=
n*

45,000
n *  10  = 12.857
45,000  10,000

2
Rmax= = 0.1556
12.857

B5  (1  0.1556)5  $45,000 = $19,317.6


PART (B):

For Declining-Balance Model, DB model:

Bt  (1  R ) t P

where R is less than Rmax, i.e. the DDB model is a particular case of the DB model.

Bt
R  1 t
P
B5for SYD model is $19,545.5

Then:

19,545.5
R  1 5 = 0.1536
45,000

Since R<Rmax, it is possible to use it for tax purposes.

Salvage value at the end of the 10th year is equal to the Book value at this year,B10.

Bt  (1  R ) t P

B10  (1  0.1536)10  $45,000= $8,491.2

PART (C):
The recommended depreciation model is the DDB Model

which provides the lowest salvage value which is useful in calculating the taxes and
provides a realistic approximation.

I recommend the double-declining-balance model because we have to consider the


worst case which is represented by the DDB
Chapter 7

Problem 7.1:
An investment proposal, which has an initial cost of
$13,000, yields net annual receipts of $3,000 for 7 years.

A) If the minimum attractive rate of return, MARR, is 15%,


do you recommend investment in this proposal? Why? Give
at least three reasons.

B) What is the minimum net annual receipt for the proposal


to be acceptable?

C) What is the value of MARR for the proposal to break-


even with the other investment opportunities?
Solution 7.1:

A. When MARR IS 15% ,WE don't recommend


investment in this proposal.

 The three reasons:-


1. The present worth amount:

PW(i)=Fjo+A(P/A i ,n)

(1+0.15)7 −1
PW(15)= -13000+3000 * = -$518.74
0.15∗(1+0.15)7

 Since the PW(i) is negative value so the proposal not


recommend.

2. Future worth amount:

FW(i) = PW(i) (F/P i ,n)


= -518.74*(1+0.15)7= -$1379.86

 Since the FW(i) is negative value so the proposal not


recommend.

3.The payout period(POP):


13000
POP = =4.3≈5 year
3000

 Since the proposal service life is 7 year it's a long time to


pay back its first cost so the investment in this proposal
not recommend.

B. The minimum net annual receipt for the proposal to


be acceptable:
0.15(1  0.15)7
A=P(A/P i ,n)=p * i(1  i) n
=13000 * =$3214.6
(1  i ) n  1 (1  0.15)7  1

C.The value of MARR for the proposal to break-even


with the other investment opportunities.

 At the break-even point :

f1(x*)=f2(x*).
Annual income=annual cost
A=P(A/P i ,n)
𝑛
3000=13000 * (1+𝑖)
𝑖(1+𝑖)
𝑛 −1

i=13.6454

Problem 7.3:

In considering the purchase of a Japanese automobile with air


conditioning the average fuel consumption is estimated as that the car
consumes 1 litre every 10 Km (kilo meters) , when the air conditioning ,
AC , is off and it makes only 7.7 Km per liter when the AC is on . The AC
will add $900 to the first cost of the automobile and will increase its
trade in value by $250 after a service life of 5 years. The air conditioning
will be used 25% driving time and will add $100 per year to the
maintenance of the automobile. The benefit of having AC when needed
is considered to be worth 5 cents per mile. Gasoline costs 25 cents per
litre, and the interest rate is taken as 15% and the owner plans to use
the automobile for 5 years.

a) If the automobile is driven for an average of 2,000 km per month,


determine the annual cost of the AC.
b) How many miles per month must the automobile be driven for the
air conditioning AC to break-even? State clearly all the
assumptions you used.
$1.00 = 100 cents.

1 mile =1600 meters.


Solution:

sv = $250

5yrs

P=$900

a) Fuel consumption for AC:


(1/7.7 - 1/10) = 0.0299 litre /km
for fuel cost for AC:

o = (2000)(12)(0.25)(0.0299)($0.25) = $44.85
annual M&O cost = 100+$44.85 = $144.85

TEA cost = (p-sv) () + sv (i) +M&O =$650() + $250(0.15) + $144.85 =


$376.26

b) Assuming x miles /yr

x miles =1.6 km

Annual M&O cost = $(100+0.00299x)

TEA costs = $231.41 + $(100+0.00299x) = $(331.41+0.00299x)

Annual benefit = $0.05x

To break-even: TEA costs = annual benefit

$(331.41+0.00299x) = $0.05x

331.41 = 0.04701x

x = 7049.777 miles /yr


The monthly driving distance to break-even is given as x/12 = 587.481
mile

Another Solution:

a)

- AC consumption:

Fuel consumption when the AC is off  10 km / liter

Fuel consumption when the AC is on  7.7 km / liter

10  7.7  2.3 km/ liter

Liters consumed by the AC  1 2.3 liter / km  0.4347 liter / km

- AC first cost  200 K .D. recovered in 5 years.

- salvage value = 70 K .D. recovered in 5 years.

- Maintenance cost = 30 K .D. / year

- Benefit cost = 20 fils / mile  201.6 fils / km  12.5 fils / km

- Diesel cost = 40 fils / liter

70 K.D.

200 K.D. 30K.D. 30K.D. 30K.D. 30K.D. 30 K.D.


-Distance traveled each month = 1,500 km

The AC is turned on for 25% of


1,500 0.25  4,500 km / month
this distance .i.e.:

- Distance traveled per year:

12  375  4,500 km/ year

- Annual consumption cost:

0.435 liter / km 4,500 km / year 0.4 K.D. / liter  763 K.D. / year

- Annual benefit:

0.125 K.D. / km 4,500 km / year  562.5 K.D. / year


562.5 K.D.

70
K.D.

200K.D. 30+783 K.D.

TEA  200  A / P 13,5   813  562.5  70  A / F 13,5 



   
 200 0.2843 250.5  70  0.1543
 56.86  250.5  10.801
 296.559 K.D./year

b) Assuming that the car travels X km / month for AC:

0.2512 X km / month  3 X km / year


Annual consumption cost for AC:

 0.435 liter / km  3 X km / year  0.4 K .D / liter


 0.414 X K .D. / year

Annual benefit:

 0.125 K.D / km 3 X km / year  0.375 K.D / year

TEA  200  A / P 13,5   30  0.414 X   0.375 X  70


 
 A/ F 13,5
 0
 200 0.2843  30  0.039 X  70  0.1543  0
 56.86  30  0.039 X  10.801 0
 76.059  0.039 X  0
Problem (7.4):
A constructing company with MAAR of 18% is considering a purchasing of a diesel engine to
power bulldozer. The first cost of engine is $9500 and it is service life is 7 years, its hourly
fuel consumption is$7.25 and it’s associated other labor cost are $5.9/hr .the maintenance
cost is zero the first year and increases by 22% of the first cost each year there after. The
salvage value of the engine is assumed to be 33% of it is first cost at any time within it is
service life. Assuming discrete compounding and that all cost .assuming discrete
compounding and that all cost are located at the end of the year concerned.

i) If the company planes to use the engine 16 hours a day for 335 days every year
determine the economic life of the engine and it is minimum annual cost.

ii) If the company can rent a similar engine complete with its fuel, labour and other
costs for $17.5per hour determine the number of operating hours per day, X, for 335 days
per year for purchasing decision to break even with the renting decision. Under what
circumstances do you recommend renting? Why?

Solution

Cash flow:
P=$9,500

(i) The fuel cost = $7.25 per hour.

The labour cost (O) =$5.90 per hour.

The operating cost =$13.15 per hour.

The total operating cost = ($13.15) (16) (335) =$70,484.

The salvage value = (0.33) ($9,500) =3,135.

Increasing in maintenance (G) = (0.22) ($9500) = $2090.

Maintenance cost M (t):

M (t) = A  G( )  0.0  $2090( A / G 18,t )


A / G m,t

TEA COST = ( P  SV )( )  SV (i )  M & O


A / P i ,n

TEA (t) = $6,365( A / P 18,t )  $3,135(0.18)  ($2090( A / G 18,t )  70,484)


This equation can be solved by setting various values of n=1, 2, 3, 4, 5, 6, 7

.Which yields the following:


TEA(cost) N

$78,559 1

$76,072.48 2

$75,836.13 3

$76,120.35 4

$76,579.49 5

$77,100.83 6

$77,635.13 7

From the above table:

nec  3 years
The economic life: .

The minimum TEA (cost) = $75,863.13.

(ii) The minimum annual cost TEA (3) for ‘x’ hour/day is given by:

TEA(3)=$6,365 (
A / P 18, 3
)  $2090( A / G 18,3 )  (5.9  7.25)(335) x

TEA (3) = $4,787.83 + 4,405.25 x

For renting the annual cost, R, is given as:

R =$17.50(335) x=5,862.5x

*
At break even point [x = x ] and TEA (3) = R

* *
$4,787.83+$4,405.25 x =$17.50(335) x

x *  3.2855hours / day
*
Renting is recommended when x< x

So renting is recommended when it plane to use the engine for less than 3.2855 hours/day.

Problem 7.5 :

Mr. Mahir has a frozen-food warehouse and is considering an extension for the storage
capacity. This extension will have 200 linear meters of walls 5.5m high. The insulation of
these walls can be either 15mm or 25mm thick. Using the additional 10mm of insulation
thickness will add $28 per m2 to the cost of constructing the extension, but will save 3.3
tons of refrigeration. Each ton of refrigeration costs $6,700 to install and $1,950 per year in
electrical energy and maintenance cost.

Money can be invested at a nominal rate of 15%.

a) If the warehouse will be used for 12 years, what is the net annual economic advantage of
using 15mm insulation?

b) How long will it take to recover the net cost of installing the 25mm insulation?

c) What is the maximum additional cost per m² of the 25mm insulation for the two
alternatives to break even?

Solution 7.5:
a)-Cost of constructing

= $28 x (200 x 5.5) = $30,800

-The first cost of installing 3.3 tons

= 3.3 x $6,700 = $22110

A1 = $1950/year*3.3ton=$6435
A / P 15,12
A **  8690( )  $ 1603.305
0.1845

Mr. Mahir will loss ($6435 - $1603.305)  $4831.695 annually

b)
$8690
$8690  $6435( P / A15,n )  ( P / A15,n ) 
$6435
(1  i ) n  1
 1.3504
i (1  i ) n
(1.15) n  1
 1.3504  n  1.619 years
0.15(1.15) n

c)

r = maximum additional cost per m²

c)
$6435
m( A / P15.12 )  A2  $6435  m   $34878.05
0.1845
m  ( p  $22110)  p  $56988.05
p  ( 200 * 5.5) * r  r  $51.807
Problem (7.6)
The morphy corporation is now considering two plans for constructing one of its new
petrochemical plant. The two plants will result in the same income per year at all
times. Plan “X” is to build a half-size plant now, at a cost of $ 3 million and a
comparable operating cost of $450,000 per year, for the first 3 years. At the end of
the third year an additional costing $2 million will be installed to double capacity.
Costs thereafter are $660,000 per year. Plant “Y” is to be build a full-scale plant now,
at a cost of $ 3.75 million with operating costs of $ 520,000 per year for the first 2
years, and $550,000 per year thereafter.

a) If money can be invested at a nominal rate of 15% compounded continuously and


a period of 12 years ahead is the base for a comparison which plant do you
recommend for the morphy corporation?

b) What would the cost of the plan “x” be for the two plans to break-even?

c) Determine the annual operating cost of plan “y” which results in a break-even.

d) Assuming that data for plant “y” remains specified in the problem specify the
conditions under which plan “x” would be economically preferable.

Solution:

a) For plan X
For any plan to be profitable the total equivalent amount must be minimum.
The annual equivalent of the plan cost is:

𝐴1 = 3,000,000 ( 𝐴/ 𝑃 15 ,12 )

The annual equivalent of the plan operating cost is:

𝐴2 = 2,000,000 (𝑃/ 𝐹 15 ,3 ) ( 𝐴/ 𝑃 15 ,12 )

𝐴3 = 450,000 (𝑃/ 𝐴 15 ,3 ) (𝐴/ 𝑃 15 ,12 )

𝐴4 = 660,000 (𝑃/ 𝐴 15 ,9 ) (𝑃/ 𝐹 15 ,3 ) (𝐴/ 𝑃 15 ,12 )

(TEA)= 𝐴1 +𝐴2 + 𝐴3 + 𝐴4
𝐴/ 𝑃 15 ,12
𝐴1 = 3,000,000 ( 0.1939 )
𝑃/ 𝐹 15 ,3 𝐴/ 𝑃 15 ,12
𝐴2 = 2,000,000( 0.6376 )( 0.1939 )
𝑃/ 𝐴 15 ,3 𝑃/ 𝐹 15 ,3 𝐴/ 𝑃 15 ,12
𝐴3 = 450,000( 2.2392 )( 0.6376 )( 0.1939 )
𝑃/ 𝐴 15 ,9 𝐴/ 𝑃 15 ,12
𝐴4 = 660,000( 4.5773 ) ( 0.1939 )

(TEA)𝑋 =581,700 + 247,261.28 + 124,575.1781 + 585,775.3902

= 1,539,311.848 $

a) For plan Y:

The annual equivalent of the plan cost is:

𝐴1 = 3,750,000 ( 𝐴/ 𝑃 15 ,12 )

The annual equivalent of the plan operating cost is:


𝐴2 = 520,000 (𝑃/ 𝐴 15 ,2 ) (𝐴/ 𝑃 15 ,12 )

𝐴3 = 550,000 (𝑃/ 𝐴 15 ,10 ) (𝑃/ 𝐹 15 ,2 ) (𝐴/ 𝑃 15 ,12 )

(TEA)𝑌 = 𝐴1 +𝐴2 + 𝐴3
𝐴/ 𝑃 15 ,12
𝐴1 = 3,750,000 ( 0.1939 )
𝑃/ 𝐴 15 ,2 𝐴/ 𝑃 15 ,12
𝐴2 = 520,000( 1.60153 )( 0.1939 )
𝑃/ 𝐴 15 ,10 𝑃/ 𝐹 15 ,2 𝐴/ 𝑃 15 ,12
𝐴3 = 450,000( 4.8004 )( 0.7408 )( 0.1939 )

(TEA)𝑌 =727,125 + 161,479.0688 + 310,290.6746

= 1,198,894.743 $

SINCE (TEA)𝑌 <(TEA)𝑋

PLAN Y is recommended.

b) For the two plan to break even:

(TEA)𝑋 = (TEA)𝑌
𝐴/ 𝑃 15 ,12 𝑃/ 𝐹 15 ,3 𝐴/ 𝑃 15 ,12
𝑃𝑋 ∗ ( 0.1939 ) + 2,000,000( 0.63763 )( 0.1939 )+
𝑃/ 𝐴 15 ,3 𝑃/ 𝐹 15 ,3 𝐴/ 𝑃 15 ,12 𝑃/ 𝐴 15 ,9 𝐴/ 𝑃 15 ,12
450,000( 2.2392 )( 0.6376 )( 0.1939 ) + 660,000( 4.5773 ) ( 0.1939 )=
1,198,894.743 $

𝑃𝑋 = 1244367.69 $

c) For the two plan to break even:

(TEA)𝑌 = (TEA)𝑋

3,750,000 ( 𝐴/ 𝑃 15 ,12 ) + 𝐶𝑌 (𝑃/ 𝐴 15 ,12 ) (𝐴/ 𝑃 15 ,12 ) = 1,539,311.848 $

𝐶𝑌 = 8182107.7325 $
d) For plan X to be economically profitable the following variable can be changed
with interest rate and period of comparison constant at “i=15” ,” n= 12” .
𝐴/ 𝑃 15 ,12 𝑃/ 𝐹 15 ,3 𝐴/ 𝑃 15 ,12 𝑃/ 𝐴 15 ,3 𝑃/ 𝐹 15 ,3 𝐴/ 𝑃 15 ,12
𝑃𝑋 ∗ ( 0.1939 ) + 𝐶1 ( 0.63763 ) ( 0.1939 ) + 𝐶2 ( 2.2392 )( 0.6376 )( 0.1939 )
𝑃/ 𝐴 15 ,9 𝐴/ 𝑃 15 ,12
+ 𝐶3 ( 4.5773 ) ( 0.1939 ) < 1,198,899.783 $

𝑃𝑋 = 𝑓𝑖𝑟𝑠𝑡 𝑐𝑜𝑠𝑡 𝑜𝑓 𝑐𝑜𝑛𝑠𝑡𝑟𝑢𝑐𝑡𝑖𝑜𝑛 𝑝𝑙𝑎𝑛 X.

𝐶1 = 𝑎𝑑𝑑𝑖𝑡𝑖𝑜𝑛𝑎𝑙 𝑐𝑜𝑠𝑡 𝑜𝑓 𝑑𝑜𝑢𝑏𝑢𝑙𝑖𝑛𝑔 𝑡ℎ𝑒 𝑐𝑎𝑝𝑎𝑐𝑖𝑡𝑦 𝑜𝑓 𝑝𝑙𝑎𝑛 X.

𝐶2 = 𝑜𝑝𝑒𝑟𝑎𝑡𝑖𝑛𝑔 𝑐𝑜𝑠𝑡 𝑜𝑓 𝑓𝑖𝑟𝑠𝑡 𝑡ℎ𝑟𝑒𝑒 𝑦𝑒𝑎𝑟.

𝐶3 = 𝑜𝑝𝑒𝑟𝑎𝑡𝑖𝑛𝑔 𝑐𝑜𝑠𝑡 𝑜𝑓 𝑙𝑎𝑠𝑡 𝑛𝑖𝑛𝑒 𝑦𝑒𝑎𝑟𝑠 𝑜𝑓 𝑝𝑙𝑎𝑛 X.

i) First case determines 𝑃𝑋 that makes plan “X” more profitable:


𝐴/ 𝑃 15 ,12 𝑃/ 𝐹 15 ,3 𝐴/ 𝑃 15 ,12
𝑃𝑋 ∗ ( 0.1939 ) + 2,000,000( 0.63763 )( 0.1939 )+
𝑃/ 𝐴 15 ,3 𝑃/ 𝐹 15 ,3 𝐴/ 𝑃 15 ,12 𝑃/ 𝐴 15 ,9 𝐴/ 𝑃 15 ,12
450,000( 2.2392 )( 0.6376 )( 0.1939 ) + 660,000( 4.5773 ) ( 0.1939 )<
1,198,899.783 $
𝐴/ 𝑃 15 ,12
𝑃𝑋 ∗ ( 0.1939 )+ 247,261.28 + 124,575.1781 + 585,775.3902 < 1,198,894.743 $

𝑃𝑋 < 1244367.69 $

ii) second case determines 𝐶1 that makes plan “X” more profitable:
𝐴/𝑃 15,12 𝑃/𝐹 15,3 𝐴/𝑃 15,12
3,000,000 ∗ ( 0.1939 ) + 𝐶1 ( 0.63763 )( 0.1939 )+
𝐴 𝑃 𝐴
𝑃/𝐴 15,3 𝑃/𝐹 15,3 ,12 ,9 ,12
𝑃 15 𝐴 15 𝑃 15
450,000( )(
2.2392 0.6376 )( 0.1939 ) + 660,000( 4.5773 ) ( 0.1939 )<
1,198,894.743 $
𝑃/𝐹 15,3 𝐴/𝑃 15,12
581,700 + 𝐶1 ( 0.63763 )( 0.1939 )+ 124,575.1781 + 585,775.3902 <1,198,894.743 $

𝐶1 =

iii) Third case determines 𝐶2 that makes plan “X” more profitable:
𝐴/𝑃 15,12 𝑃/𝐹 15,3 𝐴/𝑃 15,12
3,000,000 ∗ ( 0.1939 ) + 2,000,000( 0.63763 )( 0.1939 )+
𝐴 𝑃 𝐴
𝑃/𝐴 15,3 𝑃𝐹 15,3 ,12 ,9 ,12
𝑃 15 𝐴 15 𝑃 15
𝐶2 ( 2.2392 )(0.6376 )( 0.1939 ) + 660,000( 4.5773 ) ( 0.1939 ) < 1,198,899.783 $
iii) Fourthcase determines 𝐶2 that makes plan “X” more profitable:
𝐴/ 𝑃 15 ,12 𝑃/ 𝐹 15 ,3 𝐴/ 𝑃 15 ,12
3,000,000 ∗ ( 0.1939 ) + 2,000,000( 0.63763 )( 0.1939 )+
𝑃/ 𝐴 15 ,3 𝑃/ 𝐹 15 ,3 𝐴/ 𝑃 15 ,12 𝑃/ 𝐴 15 ,9 𝐴/ 𝑃 15 ,12
450,000( 2.2392 )( 0.6376 )( 0.1939 ) + 𝐶3 ( 4.5773 ) ( 0.1939 )<
1,198,899.783 $

You might also like